Nurs 341 Exam 1

¡Supera tus tareas y exámenes ahora con Quizwiz!

What is the NURSES RESPONSIBILITY in regards to the INFUSION THERAPY PRESCRIPTION?

-Determine that the prescription is "APPROPRIATE" for the PATIENT -Clarify any QUESTIONS -Check for ACCURACY -Check for COMPLETENESS

What cause FLUID OVERLOAD in patients receiving ISOTONIC INFUSATE?

-FLUID "ACCUMULATES" in the EXTRACELLULAR CELLS

ISOTONIC INFUSATE PATIENT RISK

-FLUID OVERLOAD (especially older adults)

Types of INFUSION THERAPY fluid

-IV Solutions -Parenteral Nutrition -Blood & Blood Products -Drugs

What does infusion therapy maintain?

-Maintain FLUID BALANCE -Maintain ELECTROLYTE BALANCE -Maintain ACID BASE BALANCE

HYPERTONIC INFUSATE PATIENT RISK:

-PHLEBITIS -INFILTRATION

HYPOTONIC INFUSATE PATIENT RISK

-PHLEBITIS -INFILTRATION

What is "REQUIRED" before INFUSION THERAPY "BEGINS"

-PRESCRIPTION

What is an example of HYPERTONIC INFUSATE

-Parental Nutrition

7 Major Types of VAD's

-Short Cath -Midline Cath -PICC -Nontunneled Percutaneous Central Cath -Implanted Ports -Hemodialysis Cath

How are IV SOLUTIONS "classified"?

-TONICITY (concentration) -PH

ADVERSE DRUG EFFECT

-UNDESIRABLE AND IMMEDIATE SERIOUS REACTION to the administration of IV MEDS ***(BCMA) HELP REDUCE THIS EFFECT***

ISOTONIC INFUSATE

-Water "DOES NOT" move IN OR OUT of the body cells

HYPOTONIC INFUSATE

-Water is moved "INTO" the body cells & "expands" them

HYPERTONIC INFUSATE

-Water is moved "OUT" of the body cells "INTO" the bloodstream

The CDC recommends whom to "initiate and maintain" infusion therapy?

-a SPECIALIZED team of INFUSION NURSES

Pneumothorax

-air in the pleural cavity caused by a puncture of the lung or chest wall

Why should the veins on the hand of an "older"" adult be avoided?

-loss of skin turgor -poor vein condition

Why should the veins on the "palm" side of the hand be avoided?

-median nerve is located close to the veins -more painful -difficult to stabilize

A patient reporting feeling "pins & needles" or numbness during venipuncture, what is indicated?

-nerve puncture

A dilution no greater than ____ of potassium to ____of solution is recommended for IV administration. The maximum recommended infusion rate is ______; this rate is never to exceed _____ under any circumstances.

1 mEq 10 mL 5 to 10 mEq/hr 20 mEq/hr

Prevent accidental overdose of IV potassium by checking and re-checking the concentration of potassium in the IV solution, ensuring that the maximum concentration is no greater than

1 mEq/10 mL of solution.

Magnesium (Mg2+) blood serum range

1.3-2.1 mEq/L

A desirable LDL-C level is one below _______ for healthy people and below _______for those diagnosed with CVD or who are diabetic.

100 mg/dL 70 mg/dL

Normal range for PT

11-12.5 seconds

Patients with diabetes and heart disease should have a blood pressure below

130/90

Sodium (Na+) blood serum range

136-145 mEq/L

Normal range for platelets

150,000 - 400,000 mm3

A 22-year-old client is seen in the emergency department (ED) with acute right lower quadrant abdominal pain, nausea, and rebound tenderness. It appears that surgery is imminent. What gauge catheter does the ED nurse choose when starting this client's intravenous (IV) solution? A. 24 B. 22 C. 18 D. 14

18

A nurse is caring for a group of hospitalized clients. Which client is at greatest risk for infection and sepsis? A. 18-year-old who had an emergency splenectomy B. 22-year-old with recently diagnosed sickle cell anemia C. 38-year-old with hemolytic anemia D. 40-year-old alcoholic with liver disease

18-year-old who had an emergency splenectomy

Total serum cholesterol levels should be below

200 mg/dL. Healthy range is 122-200 mg/dL

What is the ratio of bicarbonate to carbonic acid?

20:1

Normal range for HCO3

21-28

Normal Serum Osmolarity for adults

270-300 mOsm/L

Which client does the nurse assign as a roommate for the client with aplastic anemia? A. 23-year-old with sickle cell disease who has two draining leg ulcers B. 28-year-old with glucose-6-phosphate deficiency (G6PD) anemia who is receiving mannitol (Osmitrol) C. 30-year-old with leukemia who is receiving induction chemotherapy D. 34-year-old with idiopathic thrombocytopenia who is taking steroids

28-year-old with glucose-6-phosphate deficiency (G6PD) anemia who is receiving mannitol (Osmitrol)

Phosphorus (P) blood serum range

3.0-4.5 mg/dL

Potassium (K+) blood serum range

3.5-5.0 mEq/L

USUAL RANGE of PH of IV SOLUTION

3.5-6.2

Normal range for PTT

30-40 seconds

The body functions best when the osmolarity of the fluids in all body fluid spaces is close to ____.

300 mOsm/L

Which client is at greatest risk for having a hemolytic transfusion reaction? A. 34-year-old client with type O blood B. 42 year-old client with allergies C. 58 year-old immune suppressed client D. 78-year-old client

34-year-old client with type O blood

Normal range for triglycerides

35-160

Normal range for PaCO2

35-45

What is the normal range of RBCs?

4,200,000 to 5,400,000/mm3

WHat is the normal cardiac output?

4-8 liters/ min.

A client with hyperkalemia is being treated with drugs to improve the condition. Which potassium level indicates that therapy is effective? A. 7.6 mEq/L B. 5.6 mEq/L C. 4.6 mEq/L D. 2.6 mEq/L

4.6 mEq/L

A desirable HDL-C level is

40 mg/dL or above

The minimum amount of urine per day needed to excrete toxic waste products is ________.

400-600 mL

An RN from pediatrics has "floated" to the medical-surgical unit. Which client is assigned to the float nurse? A. 42-year-old with sickle cell disease receiving a transfusion of packed red blood cells B. 50-year-old with pancytopenia needing assessment of risk factors for aplastic anemia C. 55-year-old with folic acid deficiency anemia caused by alcohol abuse who needs counseling D. 60-year-old with newly diagnosed polycythemia vera who needs teaching about the disease

42-year-old with sickle cell disease receiving a transfusion of packed red blood cells

Which client is at increased risk for fluid and electrolyte imbalance? Select all that apply. A. 22-year-old pregnant woman in her third trimester B. 24-year-old male athlete C. 45-year-old man on diuretics D. 47-year-old man traveling to South America in summer E. 76-year-old bedridden woman

45-year-old man on diuretics 47-year-old man traveling to South America in summer 76-year-old bedridden woman

After receiving change-of-shift report, which client does the RN assess first? A. 26-year-old with nausea and vomiting who complains of dizziness when standing B. 36-year-old with a nasogastric (NG) tube who has dry oral mucosa and is complaining of thirst C. 46-year-old receiving IV diuretics whose blood pressure is 95/52 mm Hg D. 56-year-old with normal saline infusing at 150 mL/hr whose hourly urine output has been averaging 75 mL

46-year-old receiving IV diuretics whose blood pressure is 95/52 mm Hg

The nurse is caring for a client who is receiving rituximab (Rituxan) for treatment of lymphoma. It is essential for the nurse to observe for which side effect? A. Alopecia B. Allergy C. Fever D. Chills

Allergy

The nurse anticipates administering which medication to treat hyperuricemia associated with tumor lysis syndrome (TLS)? A. Recombinant erythropoietin (Procrit) B. Allopurinol (Zyloprim) C. Potassium chloride D. Radioactive iodine 131

Allopurinol (Zyloprim)

What are HYPERTONIC INFUSATES used for?

-"CORRECT" fluid, electrolyte and acid base imbalances

What does the INFUSION NURSE do?

-"Develop" evidenced based polices & procedures -"Insert" several types of peripheral & Cardiovascular CATHS -"Monitor" patient outcomes -"Educate" staff, patients & families -"Consult" on product selection & purchasing decisions

SENTINEL EVENT:

-ACUTE HEMOLYTIC TRANSFUSION REACTION due to "IMCOMPATIBLE BLOOD TRANSFUSION"

What are the PATIENT RISKS with insertions in the HANDS & FEET?

-DVT -INFILTRATION

What are 5 important facts about infusion therapy?

-Delivery of parenteral meds & fluids -Via wide variety of catheters -In wide variety of locations -Used in multiple procedures -Delivered in all health care settings

What leukocytes are involved in antibody-mediated immunity?

B-lymphocytes Plasma Cell Memory Cell

The nurse suspects metastasis from left breast cancer to the thoracic spine when the client has which symptom? A. Vomiting B. Back pain C. Frequent urination D. Cyanosis of the toes

Back pain

If you have a pt that has a valve disorder, what would you tell them about exercise?

Balance your exercise. If you are getting tired or short of breath, you need to stop. Must keep the balance of oxygen the same

A 68-year-old man is admitted to the hospital with dehydration. Initial laboratory results include a potassium level of 2.7 mEq/L. He has a history of atrial fibrillation, congestive heart failure (CHF), and hypertension. His medications are digoxin (Lanoxin), chlorothiazide (Diuril), and potassium supplements. In time, he recovers from his dehydration and low potassium levels. He says to the nurse, "I would like to take fewer medications and eat foods that contain high amounts of potassium." What foods does the nurse recommend? Select all that apply. A. Apples B. Bananas C. Broccoli D. Oranges E. Spinach

Bananas Broccoli Oranges Spinach

B-lymphocyte

Becomes sensitized to foreign cells and proteins

What two leukocytes attack grafts and transplanted organs?

Cytotoxic/lytic T-Cells Natural Killer Cells

A nurse is caring for a client who is receiving intravenous (IV) magnesium sulfate (MgSO4). Which assessment parameter is critical? A. 24-hour urine output B. Asking the client about feeling depressed C. Hourly deep tendon reflexes (DTRs) D. Monitoring of serum calcium levels

Hourly deep tendon reflexes (DTRs)

A 32-year-old man is recovering from a sickle cell crisis. His discomfort is controlled with pain medications, and he is to be discharged. What medication does the nurse expect to be prescribed for him before his discharge? A. Heparin (Heparin) B. Hydroxyurea (Droxia) C. Tissue plasminogen activator (t-PA) D. Warfarin (Coumadin)

Hydroxyurea (Droxia)

A nurse is transfusing 2 units of packed red blood cells (PRBCs) to a postoperative client. What post-transfusion electrolyte imbalance does the nurse want to rule out? A. Hypercalcemia B. Hyperkalemia C. Hypomagnesemia D. Hyponatremia

Hyperkalemia

What is the most common disease that can cause increasing afterload?

Hypertension

An 82-year-old client with anemia is requested to receive 2 units of whole blood. Which assessment finding causes the nurse to discontinue the transfusion because it is unsafe for the client? Select all that apply. A. Capillary refill less than 3 seconds B. Decreased pallor C. Flattened superficial veins D. Hypertension E. Hypotension F. Rapid, bounding pulse

Hypertension Hypotension Rapid, bounding pulse

IF you got a muscle cramp after giving a diuretic, what is this caused by?

Hypokalemia

PHLEBITIS:

INFLAMMATION OF A VEINS

What is the most common route of infusion therapy

IV (Intravenous) Therapy

A client who is to undergo cardiac catheterization should be taught which essential information? A. Monitor the pulses in your feet when you get home. B. Keep your affected leg straight for at least 8 hours. C. Do not take your blood pressure medications on the day of the procedure. D. Take oral hypoglycemic with a sip of water on the morning of the procedure.

Keep your affected leg straight for at least 8 hours.

The nurse teaches a morbidly obese client who has chosen gastric bypass surgery to promote weight loss that he will need to perform monitoring to detect what disturbance consistent with rapid weight loss associated with this procedure? A. Ketosis B. Hypoxemia C. Urinary retention D. Insufficient ventilation

Ketosis

Etiology of Secondary Hypertension

Kidney disease Primary aldosteronism Pheochromocytoma Cushing's disease Coarctation of the aorta Brain tumors Encephalitis Pregnancy Drugs: Estrogen (e.g., oral contraceptives) Glucocorticoids Mineralocorticoids Sympathomimetics

The nurse is providing community education about prevention of atherosclerosis-related diseases. Which risk factors should the nurse include in the presentation? Select all that apply. A. LDL cholesterol of 160 mg B. Smoking C. Aspirin (ASA) consumption D. Type 2 diabetes E. Vegetarian diet

LDL cholesterol of 160 mg Smoking Type 2 diabetes

INFILTRATION:

LEAKAGE OF A NONVESICANT IV SOLUTION/MED INTO THE EXTRAVASCULAR TISSUE

The nurse who is conducting an assessment of fluid and electrolyte balance in an older female patient should be certain to question the patient about: A. Laxative use B. Sleep patterns C. Changes in taste D. Dry mouth

Laxative use

The nurse is caring for an adult client with Down syndrome who reports fatigue and shortness of breath. Which type of cancer has been identified in clients with Down syndrome? A. Breast cancer B. Colorectal cancer C. Malignant melanoma D. Leukemia

Leukemia

A premenopausal client diagnosed with breast cancer will be receiving hormonal therapy. The nurse anticipates that the physician will request which medication for this client? A. Anastrazole (Arimdex) B. Fulvestrant (Faslodex) C. Leuprolide (Lupron) D. Trastuzumab (Herceptin)

Leuprolide (Lupron)

Risk factors for atherosclerosis

Low HDL-C, High LDL-C, Increased triglycerides, Genetic predisposition, Diabetes mellitus, Obesity, Sedentary lifestyle, Smoking, Stress, African-American or Hispanic ethnicity, Older adult

The nurse is teaching a group of clients about cancers related to tobacco or tobacco smoke. Identify the common cancers related to tobacco use. Select all that apply. A. Cardiac cancer B. Lung cancer C. Cancer of the tongue D. Skin cancer E. Cancer of the larynx

Lung cancer Cancer of the tongue Cancer of the larynx

A client with a high genetic risk for breast cancer asks the nurse about options for prevention and early detection. Which option for prevention and early detection is the option of choice? A. Breast self-examination (BSE) beginning at 20 years of age B. Hormone replacement therapy combining estrogen and progesterone C. Magnetic resonance imaging (MRI) and mammography every year beginning at age 30 D. Prophylactic mastectomy

Magnetic resonance imaging (MRI) and mammography every year beginning at age 30

To decrease the risk of acid-base imbalance, what goal must the client with diabetes mellitus strive for? A. Checking blood glucose levels once daily B. Drinking 3 L of fluid per day C. Eating regularly, every 4 to 8 hours D. Maintaining blood glucose level within normal limits

Maintaining blood glucose level within normal limits

Albumin

Maintains osmotic pressure of the blood, preventing the plasma from leaking into the tissues

Which client has the highest risk for cardiovascular disease? A. Man who smokes and whose father died at 49 from myocardial infarction (MI) B. Woman with abdominal obesity who exercises three times per week C. Woman with diabetes whose high-density lipoprotein (HDL) cholesterol is 75 D. Woman who is sedentary who reports four episodes of strep throat

Man who smokes and whose father died at 49 from myocardial infarction (MI)

The nurse is caring for a client with an abdominal aortic aneurysm (AAA). Which finding in the history does the nurse recognize as a risk factor for aneurysm formation? A. Peptic ulcer disease B. Deep vein thrombosis (DVT) C. Osteoarthritis D. Marfan syndrome

Marfan syndrome

The nurse is caring for a client with an arterial line. How does the nurse recognize that the client is at risk for insufficient perfusion of body organs? A. Right atrial pressure is 4 mm Hg. B. Mean arterial pressure is 58 mm Hg. C. Pulmonary artery wedge pressure (PAWP) is 7 mm Hg. D. PO2 is reported as 78 mm Hg.

Mean arterial pressure is 58 mm Hg

A client with mild hypokalemia caused by diuretic use is discharged home. The home health nurse delegates which of these interventions to the home health aide? A. Assessment of muscle tone and strength B. Education about potassium-rich foods C. Instruction on the proper use of drugs D. Measurement of the client's urine output

Measurement of the client's urine output

Types of Phebitis

Mechanical Bacterial Chemical

The nurse is caring for a critically ill client with septic shock. The serum lactate level is 6.2. For which of the following acid-base disturbances should the nurse assess? A. Metabolic acidosis B. Metabolic alkalosis C. Respiratory acidosis D. Respiratory alkalosis

Metabolic acidosis

A client admitted to the intensive care unit (ICU) is expected to remain for 3 weeks. The nurse has orders to start an IV. Which vascular access device does the nurse choose for this client? A. Midline catheter B. Peripherally inserted central catheter (PICC) C. Short peripheral catheter D. Tunneled central catheter

Midline catheter

A client who takes corticosteroids daily for rheumatoid arthritis requires insertion of an IV catheter to receive IV antibiotics for 5 days. Which type of IV catheter does a nurse teach the new graduate nurse to use for this client? A. Midline catheter B. Nontunneled percutaneous central catheter C. Peripherally inserted central catheter D. Short peripheral catheter

Midline catheter

Which action can the same-day surgery charge nurse delegate to an experienced unlicensed assistive personnel (UAP) who is helping with the care of a client who is having a breast biopsy? A. Assess anxiety level about the surgery. B. Monitor the vital signs after surgery. C. Obtain data about breast cancer risk factors. D. Teach about postoperative routine care.

Monitor the vital signs after surgery.

Decreases anxiety, dilates the vasculature, and redues myocardial consumption in the acute stage

Morphine

MOTOR AND SENSORY DEFICITS

Motor and sensory deficits occur when cancers invade bone or the brain or compress nerves. Tumors can also cause severe pain.

A nurse is assessing a client with hyponatremia. Which finding requires immediate action? A. Diminished bowel sounds B. Heightened acuity C. Muscular weakness D. Urine output of 35 mL/hr

Muscular weakness

The client receiving chemotherapy will experience the lowest level of bone marrow activity and neutropenia during which period? A. Peak B. Trough C. Nadir D. Adjuvant

Nadir

A client is admitted to the nursing unit with a diagnosis of hypokalemia. Which assessment does the nurse complete first? A. Auscultating bowel sounds B. Checking deep tendon reflexes (DTRs) C. Determining the level of consciousness (LOC) D. Obtaining a pulse oximetry reading

Obtaining a pulse oximetry reading

A nurse is caring for a client who is in sickle cell crisis. What action does the nurse do first? A. Applies cool compresses to the client's forehead B. Encourages the client's use of two methods of birth control C. Increases food sources of iron in the client's diet D. Provides pain medications as needed

Provides pain medications as needed

An RN is caring for a client who is severely dehydrated. Which nursing action can be delegated to unlicensed assistive personnel (UAP)? A. Consulting with a health care provider about a client's lab results B. Infusing 500 mL of normal saline over 60 minutes C. Monitoring IV fluid to maintain the drip rate at 75 mL/hr D. Providing oral care every 1 to 2 hours

Providing oral care every 1 to 2 hours

Which intervention will be most helpful for the client with mucositis? A. Administering a biological response modifier B. Encouraging oral care with commercial mouthwash C. Providing oral care with a disposable mouth swab D. Maintaining NPO until the lesions have resolved

Providing oral care with a disposable mouth swab

A 77-year-old woman is brought to the emergency department by her family after she has had diarrhea for 3 days. The family tells a nurse that she has not been eating or drinking well, but that she has been taking her diuretics for congestive heart failure (CHF). She is receiving lactated Ringer's solution IV for rehydration. What clinical manifestations does the nurse monitor during rehydration of the client? Select all that apply. A. Blood serum glucose B. Pulse rate and quality C. Urinary output D. Urine specific gravity levels

Pulse rate and quality Urinary output Urine specific gravity levels

What type of blood cells does bone marrow produce?

RBC WBC Platelets

The client is a 69-year-old woman with uncontrolled diabetes, polyuria, and a blood pressure of 86/46. Which staff member is assigned to care for her? A. LPN/LVN who has floated from the hospital's long-term care unit B. LPN/LVN who frequently administers medications to multiple clients C. RN who has floated from the intensive care unit D. RN who usually works as a diabetic educator

RN who has floated from the intensive care unit

A severely dehydrated client requires a rapid infusion of normal saline and needs a midline IV line placed. Which staff member does the emergency department (ED) charge nurse assign to complete this task? A. RN who is certified in the administration of oral and infused chemotherapy medications B. RN with 2 years of experience in the emergency department who is skilled at insertion of short peripheral catheters C. RN with 10 years of experience on a medical-surgical unit who has cared for many clients requiring IV infusions D. RN with certified registered nurse infusion (CRNI) certification who is assigned to the emergency department for the day

RN with certified registered nurse infusion (CRNI) certification who is assigned to the emergency department for the day

The client who has recently had breast cancer surgery requests a volunteer to visit her home to help with recovery. Which community resource will the nurse recommend? A. National Breast Cancer Coalition B. Reach for Recovery C. Susan G. Komen for the Cure D. Young Survival Coalition

Reach for Recovery

What type of cells produce hemoglobin (Hgb)?

Red Blood Cells

The outpatient client is receiving photodynamic therapy. Which environmental factor is a priority for the client to adjust for protection? A. Storing drugs in dark locations at room temperature B. Wearing soft clothing C. Wearing a hat and sunglasses when going outside D. Reducing all direct and indirect sources of light

Reducing all direct and indirect sources of light

The large-breasted client reports discomfort, backaches, and fungal infections because of her excessive breast size. The nurse plans to provide information to the client about which breast treatment option? A. Augmentation B. Compression C. Reconstruction D. Reduction mammoplasty

Reduction mammoplasty

GENERAL DISEASE-RELATED CONSEQUENCES OF CANCER

Regardless of treatment type, cancer always affects a person's physical and psychological functioning. Most people fear cancer and cancer treatment and consider a cancer diagnosis to involve suffering and death.

A client who has just been discharged from the hospital after a modified radical mastectomy is referred to a home health agency. Which nursing action will be most appropriate to delegate to an experienced home health aide? A. Assessing the safety of the home environment B. Developing a plan to decrease lymphedema risk C. Monitoring pain level and analgesic effectiveness D. Reinforcing the guidelines for hand and arm care

Reinforcing the guidelines for hand and arm care

Basophil

Releases histamine and heparin in areas of tissue damge

Which of these does the nurse recognize as the goal of palliative surgery for the client with cancer? A. Cure of the cancer B. Relief of symptoms or improved quality of life C. Allowing other therapies to be more effective D. Prolonging the client's survival time

Relief of symptoms or improved quality of life

Memory Cell

Remains sensitized to a specific antigen and can secrete increased amounts of immunoglobulins specific to the antigen

The professional nurse and the nursing student are caring for a group of clients with hypertension. Which of these problems identified by the nursing student correctly identifies the client at risk for secondary hypertension? A. Psychiatric disturbance B. High sodium intake C. Physical inactivity D. Renal failure

Renal failure

Plasma Cell

Secretes immunoglobulins in response to the presence of a specific antigen

Cytotoxic/cytolytic T-cell

Selectively attacks and destroys non-self cells, including virally infected cells,k grafts, and transplanted organs

A 72-year-old client admitted with fatigue and dyspnea has elevated levels of all of these laboratory results. Which finding is consistent with acute coronary syndrome (ACS) and should be communicated immediately to the physician? A. White blood cell (WBC) count B. Low-density lipoproteins C. Serum troponin I level D. C-reactive protein

Serum troponin I level

What would you expect to have before recieving a cardioversion?

Short acting Sedative

Which symptom reported by the client who has had a total hip replacement requires emergency action? A. Localized swelling of one of the lower extremities B. Positive Homans' sign C. Shortness of breath and chest pain D. Tenderness and redness at the IV site

Shortness of breath and chest pain

Which vascular assessment technique by the student nurse requires intervention by the supervising nurse? A. Measuring capillary refill in the fingertips B. Assessing pedal pulses by Doppler C. Measuring blood pressure in both arms D. Simultaneously palpating the bilateral carotids

Simultaneously palpating the bilateral carotids

The nurse teaches a client who has had a myocardial infarction (MI) which information regarding diet? A. Less than 30% of the daily caloric intake should be derived from proteins. B. Use canola oil rather than palm or coconut oil. C. Consume 10 mg fiber daily. D. Work toward lowering your HDL cholesterol levels.

Use canola oil rather than palm or coconut oil.

SURGERY

Surgery to remove the malignant tissue is the oldest form of cancer treatment and was the first method used to cure cancer. Cancer surgery often involves the loss of a specific body part or its function.

A client has undergone an embolectomy for acute arterial occlusion after creation of a lower arm A-V fistula for dialysis. Which finding should the nurse report to the provider immediately? A. Swelling and tenseness in the affected area B. Incisional pain and tenderness at the surgical site C. Pink, mobile fingers D. Client who has order for heparin infusion

Swelling and tenseness in the affected area

Prehypertension

Systolic 120-139 mm Hg or diastolic 80-89 mm Hg

Stage 1: Hypertension

Systolic 140-159 mm Hg or diastolic 90-99 mm Hg

Normal Blood Pressure

Systolic <120 mm Hg and diastolic <80 mm Hg

Stage 2: Hypertension

Systolic ≥160 mm Hg or diastolic ≥100 mm Hg

What leukocytes are involved in cell-mediated immunity?

T-lymphocyte helper/inducer T-cell Cytotoxic-cytolytic T-cell Natural Killer Cell

What should you do before giving digoxin?

Take an apical pulse for 1 full minute. Hold if pulse is <60 bpm

Which information must the organ transplant nurse emphasize before each client is discharged? A. Taking immune suppressant medications increases your risk for cancer and the need for screenings. B. You are at increased risk for cancer when you reach 60 years of age. C. Immunosuppressant medications will decrease your risk for developing cancers. D. After 6 months, you may stop immune suppressant medications, and your risk for cancer will be the same as that of the general population.

Taking immune suppressant medications increases your risk for cancer and the need for screenings.

The client with hypertension is started on verapamil (Isoptin). What teaching does the nurse provide for this client? A. Encourages foods high in potassium B. Monitors for irregular pulse C. Monitors for muscle cramping D. Teaches the client to avoid grapefruit juice

Teaches the client to avoid grapefruit juice

Which activity performed by the community health nurse best reflects primary prevention of cancer? A. Assisting women to obtain free mammograms B. Teaching a class on cancer prevention C. Encouraging long-term smokers to get a chest x-ray D. Encouraging sexually active women to get annual Papanicolaou (Pap) smears

Teaching a class on cancer prevention

What is cardiac rehab?

Telemetry monitored exercise

A 90-year-old client with hypermagnesemia is seen in the emergency department (ED). The ED nurse prepares the client for admission to which inpatient unit? A. Dialysis/Home Care B. Geriatric/Rehabilitation C. Medical-Surgical D. Telemetry/Cardiac Step-Down

Telemetry/Cardiac Step-Down

The nurse manager in a long-term care facility is developing a plan for primary and secondary prevention of colorectal cancer. Which tasks associated with the screening plan will be delegated to nursing assistants within the facility? A. Testing of stool specimens for occult blood B. Teaching about the importance of dietary fiber C. Referring clients for colonoscopy procedures D. Giving vitamin and mineral supplements

Testing of stool specimens for occult blood

Virchow's triad

The occurrence of stasis of blood flow, endothelial injury, or hypercoagulability; often associated with thrombus formation.

Which intervention will be most helpful in preventing disseminated intravascular coagulation (DIC)? A. Monitoring platelets B. Administering packed red blood cells C. Using strict aseptic technique to prevent infection D. Administering low-dose heparin therapy for clients on bedrest

Using strict aseptic technique to prevent infection

The nurse is assessing a client with mitral stenosis who is to undergo a transesophageal echocardiogram (TEE) today. Which nursing action is essential? A. Auscultate the client's precordium for murmurs. B. Teach the client about the reason for the TEE. C. Reassure the client that the test is painless. D. Validate that the client has remained NPO.

Validate that the client has remained NPO.

VAD?

Vascular Access Device

Which statement best reflects correct client education for a client with a blood pressure (BP) of 134/86? A. This blood pressure is good because it is a normal reading. B. This blood pressure indicates that the client has hypertension or high blood pressure. C. This blood pressure increases the workload of the heart; the client should consider modifying his or her lifestyle. D. This blood pressure seems a little low; the client should be further assessed for orthostatic hypotension.

This blood pressure increases the workload of the heart; the client should consider modifying his or her lifestyle.

If you were taking an ACE inhibitor, (vasotech) and BP is going down, is this good or bad and why?

This is good because ACE inhibitors dilate the arteries

The nurse is teaching a client about the purpose of electrophysiology studies (EPS). Which statement reflects the most correct teaching? A. This is a noninvasive test performed to assess your heart rhythm. B. You will receive an injection of dobutamine (Dobutrex) and will walk on a treadmill to reveal whether you have coronary artery disease. C. This is a painless test that is done to assess the structure of your heart using sound waves. D. This test evaluates you for potentially fatal cardiac rhythms.

This test evaluates you for potentially fatal cardiac rhythms.

When the nurse is counseling a 60-year-old African-American male client with all of these risk factors for lung cancer, teaching should focus most on which risk factor? A. Tobacco use B. Ethnicity C. Gender D. Increased age

Tobacco use

A nurse is planning care for a client with hypocalcemia. Which nursing action is appropriate to delegate to unlicensed assistive personnel (UAP)? A. Collaborating with the dietitian to provide calcium-rich foods for the client B. Evaluating the client's laboratory results C. Implementing Seizure Precautions for the client D. Transferring the client from the bed to a stretcher using a lift sheet

Transferring the client from the bed to a stretcher using a lift sheet

The nurse recognizes that which laboratory findings are consistent with acute coronary syndrome (ACS)? Select all that apply. A. Troponin 3.2 ng/mL B. Myoglobin of 234 mcg/L C. C-reactive protein 13 mg/dL D. Triglycerides 400 mg/dL E. Lipoprotein-a (Lp[a]) 18 mg/dL

Troponin 3.2 ng/mL Myoglobin of 234 mcg/L

The client has been admitted to the hospital with chest pain radiating down the left arm. The pain has been unrelieved by rest and antacids. Which test result best confirms that the client sustained a myocardial infarction? A. C-reactive protein of 1 mg/dL B. Homocysteine level 13 mmol/L C. Creatine kinase of 125 mg/dL D. Troponin of 5.2 ng/mL

Troponin of 5.2 ng/mL

The nurse caring for a client who has had AAA repair would be most alarmed by which finding? A. Urine output of 20 mL over 2 hours B. Blood pressure of 106/58 C. Absent bowel sounds D. +3 pedal pulses

Urine output of 20 mL over 2 hours

Lower pH values (below 7.35) mean?

acidosis is present

Assess the_____________ status of any patient who has acute respiratory acidosis or acute confusion

airway and oxygenation

Higher pH values (above 7.45) mean?

alkalosis is present

A decrease in intensity or a complete loss of a pulse in a patient with atherosclerosis may indicate .

an arterial occlusion (blockage).

The definitive test for leukemia is

an examination of cells obtained from bone marrow aspiration and biopsy.

Respiratory alkalosis is usually caused by

an excessive loss of CO2 through hyperventilation (rapid respirations).

fibrocystic breast condition (FBC) may be caused by

an imbalance in the normal estrogen to progesterone ratio.

VESICANT

an intravenous medication that causes blisters and tissue injury when it escapes into surrounding tissue

The best way to determine acid-base balance is to ?

analyze arterial blood gases (ABGs).

Malignant cells

are abnormal, serve no useful function, and are harmful to the normal body tissues; invasive, noncohesive, does not stop at tissue border; invades and destroys surrounding tissues; rapid growth; metastasizes to distant sites; not always easy to remove; can recur

Lymphomas

are cancers of the lymphoid tissues throughout the body, an abnormal overgrowth of one type of leukocyte, the lymphocyte.

promoters

are substances that promote or enhance growth of the initiated cancer cell. normal insulin and estrogen, can act as promoters and make altered cells divide more frequently.

Fibroadenomas

are the most common benign tumor during the reproductive years; oval, freely mobile, rubbery masses varying in size from smaller than 1 cm in diameter to as large as 15 cm in diameter.

The normal cells most affected by chemotherapy

are those that divide rapidly, including skin, hair, intestinal tissues, spermatocytes, and blood-forming cells.

30% of cancers diagnosed in north america

are tobacco use related

hematologic cancers

arise from blood cell forming tissues

solid tumors

arise from specific tissues

The common buffers are

bicarbonate, phosphate, and proteins.

standard treatment for the patient with leukemia

bone marrow transplant

Common sites of metastasis for Breast Cancer

bone, lung, liver, brain

Common sites of metastasis for Prostate Cancer

bone, pelvic nodes

Common sites of metastasis for Lung Cancer

brain, bone, liver, lymph nodes, pancreas

Autoimmune thrombocytopenic purpura, also called idiopathic thrombocytopenic purpura,

creates an antibody which is placed upon the surface of the platelets, making them more likely to be destroyed by macrophages.

One of the most common causes of low sodium levels is the prolonged use and overuse of _______.

diuretics

specific morphology:

each normal cell type has a distinct and recognizable appearance, size and shape.

promotion

enhancement of growth of an initiated cell

Breast augmentation surgery

enhances the size, shape, or symmetry of breasts. Most surgeries involve the implantation of saline-filled or silicone prostheses.

differentiated function:

every normal cell has at least one special function to contribute to whole-body function.

three interacting factors influence cancer development

exposure to carcinogens; genetic predisposition; immune function

A major complication of IV infusion chemotherapy is

extravasation, or the movement of the IV needle so the drug leaks into the surrounding tissues. Results can include pain, infection and tissue loss.

Most problems caused by overhydration are related to _________.

fluid volume excess in the vascular space or dilution of specific electrolytes and blood components.

ISOTONIC SOLUTIONS Osmolarity

fluids 270-300 mOsm/L same as body norm mOsm/L

HYPERTONIC SOLUTIONS Osmolarity

fluids GREATER than 300

HYPOTONIC SOLUTIONS Osmolarity

fluids LESS than 270

One indicator of hypocalcemia is a report of

frequent, painful muscle spasms ("charley horses") in the calf or foot during rest or sleep.

hypertrophy

growth that causes tissue to increase in size by enlarging each cell.

Assess _________ at least every 2 hours for any patient with an acid-base imbalance.

heart rate and rhythm

Natriuretic peptides (NPs)

hormones secreted by special cells that line the atria of the heart and the ventricles of the heart. These peptides are secreted in response to increased blood volume and blood pressure, which stretch the heart tissue. It binds to receptors in the nephrons, creating effects that are opposite of aldosterone. Kidney reabsorption of sodium is inhibited at the same time that glomerular filtration is increased, causing increased urine output. The outcome is decreased circulating blood volume and decreased blood osmolarity.

Interventions for metabolic acidosis include

hydration and drugs or treatments to control the problem causing the acidosis.

The kidneys regulate the amount of

hydrogen and bicarbonate ions that are retained or excreted by the body.

The kidneys regulate the amount of?

hydrogen and bicarbonate ions that are retained or excreted by the body.

The carbon dioxide content of a fluid is directly related to the amount of

hydrogen ions in that fluid.

Assess _______________for for patients taking potassium-sparing diuretics, such as spironolactone.

hyperkalemia (high potassium level) Like hypokalemia, an increased potassium level can also cause weakness and irregular pulse.

The problems caused by hyperphosphatemia center on the _________ that results when serum phosphorus levels increase.

hypocalcemia

The most frequent side effect associated with thiazide and loop diuretics is

hypokalemia (low potassium level). Monitor serum potassium levels, and assess for irregular pulse and muscle weakness, which may indicate hypokalemia.

Anaerobic conditions occur with

hypoxia, sepsis, and shock.

personal factors

immune function, age, genetic risk affect whether a person is likely to develop cancer.

Symptoms of FBC

include breast pain and tender lumps or areas of thickening in the breasts. The lumps are rubbery, ill defined, and commonly found in the upper outer quadrant

Peripheral vascular disease (PVD)

includes disorders that change the natural flow of blood through the arteries and veins of the peripheral circulation. It affects the legs much more frequently than the arms.

A rare but highly aggressive form of invasive breast cancer is

inflammatory breast cancer (IBC). Symptoms include swelling, skin redness, and pain in the breasts. IBC seldom presents as a lump and may not show up on a mammogram.

Problems of the hematologic system can affect many tissues and organs by

interfering with oxygenation and tissue perfusion.

The most serious complication from thrombolytic therapy is

intracerebral bleeding. Closely monitor patients for signs and symptoms of increasing intracranial pressure. A decreased level of consciousness (LOC) is the earliest indicator of this complication. Assess for other neurologic changes and report them immediately to the physician.

initiation

irreversible event that can lead to cancer development. a cancer cell is not a threat if it cannot divide, it cannot form a tumor.

Ductal ectasia

is a benign problem usually in women approaching menopause. It occurs when a breast duct dilates and its walls thicken, causing a blocked duct.

Orthostatic hypotension

is a decrease in blood pressure (20 mm Hg systolic and/or 10 mm Hg diastolic) when the patient changes position from lying to sitting.

Aplastic anemia

is a deficiency of circulating red blood cells which occurs when bone marrow fails to produce the red cells or by injury to the immature precursor cell for red blood cells, known as the pluripotent stem cell.

Sickle cell disease

is a genetic disorder that results in chronic anemia, pain, disability, organ damage, increased risk for infection, and early death.

Thrombotic thrombocytopenic purpura

is a rare autoimmune disorder in which platelets clump together abnormally and too few platelets remain in circulation.

Anemia

is a reduction in either the number of red cells, the amount of hemoglobin, or the hematocrit. It is a clinical sign, not a specific disease.

Malignant hypertension

is a severe type of elevated blood pressure that rapidly progresses. A person with this health problem usually has symptoms such as morning headaches, blurred vision, and dyspnea and/or symptoms of uremia

hypertension

is a systolic blood pressure at or above 140 mm Hg and/or a diastolic blood pressure at or above 90 mm Hg in people who do not have diabetes mellitus.

Arteriosclerosis

is a thickening, or hardening, of the arterial wall that is often associated with aging.

Leukemia

is a type of cancer with uncontrolled production of immature white blood cells which crowd the marrow and reduce production of normal blood cells.

Multiple myeloma

is a white blood cell cancer that involves a mature plasma cell, resulting in an overgrowth of B-lymphocyte plasma cells in the bone marrow that overproduces antibodies or gamma globulins.

Ductal carcinoma in situ

is an early noninvasive form of breast duct cancer. Although it should be treated to prevent it developing into an invasive breast cancer, it is not harmful at this stage, and not every woman will develop invasive disease.

Buerger's disease (thromboangiitis obliterans)

is an uncommon occlusive disease of the arteries and veins in the distal portion of the upper and lower extremities. The disease often extends into the tissues around the vessels, resulting in fibrosis and scarring that bind the artery, vein, and nerve firmly together.

Lobular carcinoma in situ

is another type of noninvasive cancer most often diagnosed incidentally during a biopsy for another problem

Teletherapy

is radiation delivered from a source outside of the patient. Because the source is external, the patient is not radioactive and is not hazardous to others.

Reduction mammoplasty

is removal of excess breast tissue and repositioning of the nipple and remaining skin flaps to produce an optimal cosmetic effect.

The most common type of inherited hemolytic anemia

is the deficiency of the enzyme glucose-6-phosphate dehydrogenase.

Infiltrating ductal carcinoma

is the most common type of invasive breast cancer.

benign tumor cells

normal cells growing in the wrong place, or at the wrong time; local, cohesive, well-defined borders, pushes tissues out of way; slow growth; encapsulated, easily removed; does not recur

Tight adherence:

normal cells make proteins that protrude from the membranes allowing them to bind closely and tightly together; fibronectin; keeps most normal tissues bound tightly to each other, exceptions are blood cells.

Spinal Cord Compression

occur either when a tumor directly enters the spinal cord or when the vertebrae collapse from tumor degradation of the bone. Tumors may begin in the spinal cord but more often spread from the lung, prostate, breast, and colon. It may cause back pain before nerve deficits occur. Neurologic problems are specific to the level of spinal compression and can lead to paralysis, which is usually permanent.

malignant- migration- metastasize

occurs because cancer cells do not bind tightly together and have many enzymes on their cell surfaces. can slip through blood vessels and other body sites. major cause of death.

Hypercalcemia

occurs most often in patients with bone metastasis. Cancer in bone causes release of calcium into the bloodstream. Some cancers, especially in the lung, head and neck, kidney, or lymph nodes, secrete parathyroid hormone, causing bone to release calcium. Decreased mobility and dehydration worsen hypercalcemia.

Superior Vena Cava Syndrome

occurs when the SVC is compressed or obstructed by tumor growth or by clots in the vessel. SVC compression is painful and life threatening and occurs most often in patients with lymphomas, lung cancer, and breast cancer. The manifestations result from the blockage of venous return from the head, neck, and upper trunk.

Incomplete breakdown of glucose

occurs whenever cells metabolize under anaerobic conditions, forms lactic acid.

Four processes can result in metabolic acidosis:

overproduction of hydrogen ions, underelimination of hydrogen ions, underproduction of bicarbonate ions, and overelimination of bicarbonate ions.

mitotic index

percentage of actively dividing cells within a tumor.

To test for Trousseau's sign,

place a blood pressure cuff around the upper arm, inflate the cuff to greater than the patient's systolic pressure, and keep the cuff inflated for 1 to 4 minutes. Under these hypoxic conditions, a positive sign occurs when the hand and fingers go into spasm in palmar flexion

Hematologic Disorders result in

poor function, decreased numbers, or excess of red blood cells.

Vitamin B12 deficiency results from

poor intake of foods containing vitamin B12 or problems such as small bowel resection, diverticula, tapeworm, or overgrowth of intestinal bacteria

Check the serum _______ levels for any patient who has acidosis

potassium

The four types of autologous blood transfusions are

preoperative autologous blood donation, acute normovolemic hemodilution, intraoperative autologous transfusion, and postoperative blood salvage.

Disseminated intravascular coagulation (DIC)

problem with the blood-clotting process; is triggered by many severe illnesses, including cancer. In patients with cancer, it is often is caused by gram-negative sepsis, by the release of thrombin or thromboplastin (clotting factors) from cancer cells, or by blood transfusions. Extensive, abnormal clotting occurs throughout the small blood vessels of patients with DIC. This widespread clotting uses up the existing clotting factors and platelets. This process is followed by extensive bleeding.

Antidiuretic hormone (ADH)

produced in the brain and stored in the posterior pituitary gland. Released from the posterior pituitary gland is controlled by the hypothalamus in response to changes in blood osmolarity. It acts directly on kidney tubules and collecting ducts, making them more permeable to water. As a result, more water is reabsorbed by these tubules and returned to the blood, decreasing blood osmolarity by making it more dilute.

The purpose of cancer treatment is to

prolong survival time or improve quality of life.

Cancer surgery may be used for

prophylaxis, diagnosis, cure, control, palliation, determination of therapy effectiveness, and reconstruction.

A major objective in caring for the patient with leukemia is

protection from infection. Frequent, thorough handwashing is of the utmost importance.

Although physical nursing care needs of the patient having surgery for cancer are similar to those related to other surgeries, two additional priority care needs are

psychosocial support and optimal return of function.

Any patient with fluid overload, regardless of age, is at risk for these complications.

pulmonary edema and heart failure

The respiratory system's response in acid-base balance is

rapid—efforts occur within seconds to minutes.

The problems caused by hyponatremia involve two mechanisms____________.

reduced excitable membrane depolarization and cellular swelling.

Most manifestations of hypermagnesemia occur as a result of __________.

reduced membrane excitability.

lymphatic spread

related to the number structure and locatoin of lymh nodes and vessels, primary sites that are rich in lymphatics have more arly metastatic spread than areas with few lymphatics.

Respiratory acidosis is caused by four types of problems:

respiratory depression, inadequate chest expansion, airway obstruction, and reduced alveolar-capillary diffusion.

Respiratory acidosis results from only one process:

retention of CO2, causing increased production of free hydrogen ions.

secondary prevention

routine screening, gene therapy in the future.

secondary prevention

screening strategies to detect cancer early, at time when cure or control is more likely

The effects of hypomagnesemia are caused by increased membrane excitability and the accompanying __________

serum calcium and potassium imbalances. Excitable membranes, especially nerve cell membranes, may depolarize spontaneously.

The manifestations of chronic hypophosphatemia are most evident in the _________

skeletal system. Bone density is decreased, which leads to fractures and changes in bone shape. These changes are caused by the bone calcium loss that occurs with hypophosphatemia.

cancers are divided into two major categories:

solid and hematologic.

Renal mechanisms are

stronger for regulating acid-base balance but take longer than chemical and respiratory mechanisms to completely respond.

A base is ?

substance that binds free hydrogen ions in solution

carcinogens

substances that change the activity of a cell so that the cell becomes a cancer cell.

Acids are ?

substances that release hydrogen ions when dissolved in water (H2O).

PAD is a result of

systemic atherosclerosis.

To test for Chvostek's sign,

tap the face just below and in front of the ear (over the facial nerve) to trigger facial twitching of one side of the mouth, nose, and cheek

Two types of delivery are used for cancer therapy depending on the patient's general health and on the size and location of the tumor to be irradiated.

teletherapy & brachytherapy

doubling time

the amount of time it takes for a tumor to double in size

Abnormal pH interferes with many normal physiologic functions, including:

the function of hormones and enzymes, the distribution of other electrolytes, activity of the heart, nerves, muscles, and GI tract, the effectiveness of many drugs.

If a lung problem causes retention of carbon dioxide

the healthy kidney compensates by increasing the amount of bicarbonate that is produced and retained.

Chemical buffers

the immediate way that acid-base imbalances are corrected.

Hematologic problems result from

the impaired production, impaired function, or the abnormal destruction of any type of blood cell.

Fibrocystic changes of the breast involve

the lobules, ducts, and stromal tissues and occur most often in premenopausal women between 20 and 50 years of age.

The more hydrogen ions present______

the more acidic the fluid

The fewer hydrogen ions present______

the more alkaline the fluid

primary tumor

the original tumor; usually identified by the tissue from which it arose such as breast or lung cancer.

Statin drugs are discontinued if

the patient has muscle cramping or elevated liver enzyme levels. Some patients also report abdominal bloating, flatulence, diarrhea, and/or constipation as side effect of these drugs.

Compensation

the process in which the body uses its three regulatory mechanisms (chemical, respiratory, and renal) to correct for changes in the pH of body fluids.

malignant transformation

the process of changing a normal cell into a cancer cell

Whenever the CO2 level changes, the pH changes to ______________.

the same degree in the opposite direction since carbon dioxide is the most changeable component of carbonic acid.

Oxygen Dissociation

the transfer of O2 from hemoglobin to tissues

Chemotherapy,

the treatment of cancer with chemical agents, has a major role in cancer therapy to cure and to increase survival time.

latency period

time between a cells initiation and the development of an overt tumor, can range from months to years.

TNM

tumor, node, metastasis; describes the anatomic extent of cancers, not for leukemia or lymphomas. useful for prognosis and comparison of treatment results

When lifting or moving a patient with fragile bones,

use a lift sheet rather than pulling the patient.

primary prevention

use of strategies to prevent the actual occurrence of cancer. most effective with a known cause

Raynaud's phenomenon is caused by

vasospasm of the arterioles and arteries of the upper and lower extremities, usually unilaterally. Raynaud's disease occurs bilaterally.

oncoviruses

viruses that cause cancer

Syndrome of Inappropriate Antidiuretic Hormone (SIADH)

water is reabsorbed to excess by the kidney and put into systemic circulation. The retained water dilutes blood serum sodium levels. Mild manifestations include weakness, muscle cramps, loss of appetite, and fatigue.

Insensible water loss

water loss from the skin, lungs, and stool that cannot be controlled

Incomplete breakdown of fatty acids

when large amounts of fatty acids are being metabolized, forms ketoacids.

A client is seen in the emergency department (ED) with pain, redness, and warmth of the right lower arm. The client was in the ED last week after an accident at work. On the day of the injury, the client was in the ED for 12 hours receiving IV fluids. On close examination, the nurse notes the presence of a palpable cord 1 inch in length and streak formation. How does the nurse classify this client's phlebitis? A. Grade 1 B. Grade 2 C. Grade 3 D. Grade 4

Grade 3

The nurse is caring for a client who has taken a large quantity of furosemide (Lasix) to promote weight loss. The nurse anticipates the finding of which acid-base imbalance? A. PO2 of 78 mm Hg B. HCO of 34 mEq/L C. PCO2 of 56 mm Hg D. pH of 7.31

HCO of 34 mEq/L

The nurse in the cardiology clinic is reviewing teaching about hypertension, provided at the last appointment. Which actions by the client indicate that teaching has been effective? Select all that apply. A. Has maintained a low-sodium, no-added-salt diet B. Has lost 3 pounds since last seen in the clinic C. Cooks food in palm oil to save money D. Exercises once weekly E. Has cut down on caffeine

Has maintained a low-sodium, no-added-salt diet Has lost 3 pounds since last seen in the clinic Has cut down on caffeine

A nurse is assessing the nutritional status of a client with anemia. How does the nurse obtain information about the client's diet? A. Asks the client to rate his or her diet on a scale of 1 (poor) to 10 (excellent) B. Determines who prepares the client's meals and plans an interview with him or her C. From a prepared list, finds out the client's food preferences D. Has the client write down everything he or she has eaten for the past week

Has the client write down everything he or she has eaten for the past week

Globulins

Have many functions, such as transporting other substance ans as antibodies protecting the body against infection

What are the most obvious changes that occur with dehydration?

Heart rate increases in an attempt to maintain blood pressure with less blood volume. Peripheral pulses are weak, difficult to find, and easily blocked with light pressure. The blood pressure also decreases, as does the pulse pressure, with a greater decrease in the systolic blood pressure. Skin turgor is poor, with the tent remaining for minutes after pinching the skin, and no skin depressions occur with gentle pressure. The skin is dry and scaly.

The _____ needs ______ to carry_____

Heme Iron Oxygen

The globin portion of ________ carries_______.

Hemoglobin Carbon Dioxide

A nurse instructs an older adult client to increase intake of dietary potassium when the client is prescribed which classification of drugs? A. Alpha antagonists B. Beta blockers C. Corticosteroids D. High-ceiling (loop) diuretics

High-ceiling (loop) diuretics

Limited cell division of a normal cell:

normal cells divide (mitosis) only for 1. develop normal tissue or 2. replace lost or damaged normal tissue.

A nurse is teaching a client about induction therapy for acute leukemia. Which client statement indicates a need for additional education? A. "After this therapy, I will not need to have any more." B. "I will need to avoid people with a cold or flu." C. "I will probably lose my hair during this therapy." D. "The goal of this therapy is to put me in remission."

"After this therapy, I will not need to have any more."

A nurse is teaching a client with newly diagnosed anemia about conserving energy. What does the nurse tell the client? Select all that apply. A. "Allow others to perform your care during periods of extreme fatigue." B. "Drink small quantities of protein shakes and nutritional supplements daily." C. "Perform a complete bath daily to reduce your chance of getting an infection." D. "Provide yourself with four to six small, easy-to-eat meals daily." E. "Space your care activities in groups to conserve your energy." F. "Stop the activity when shortness of breath or palpitations are present."

"Allow others to perform your care during periods of extreme fatigue." "Drink small quantities of protein shakes and nutritional supplements daily." "Provide yourself with four to six small, easy-to-eat meals daily." "Stop the activity when shortness of breath or palpitations are present."

A distant family member arrives to visit a female client recently diagnosed with leukemia. The family member asks, "What should I say to her?" Which responses does the nurse suggest? Select all that apply. A. "Ask her how she is feeling." B. "Ask her if she needs anything." C. "Tell her to be brave and to not cry." D. "Talk to her as you normally would when you haven't seen her for a long time." E. "Tell her what you know about leukemia."

"Ask her how she is feeling." "Ask her if she needs anything." "Talk to her as you normally would when you haven't seen her for a long time."

A hematology unit is staffed by RNs, LPNs/LVNs, and unlicensed assistive personnel (UAP). When the nurse manager is reviewing documentation of staff members, which entry indicates that the staff member needs education about his or her appropriate level of responsibility and client care? A. "Abdominal pain relieved by morphine 4 mg IV; client resting comfortably and denies problems. B.C., RN" B. "Ambulated in hallway for 40 feet and denies shortness of breath at rest or with ambulation. T.Y., LPN" C. "Client reporting increased shortness of breath; oxygen increased to 4 L by nasal cannula. M.N., UAP" D. "Vital signs 37.0° C, heart rate 60, respiratory rate 20, blood pressure 110/68, and oximetry 98% on room air. L.D., UAP"

"Client reporting increased shortness of breath; oxygen increased to 4 L by nasal cannula. M.N., UAP"

A nurse is assessing the endurance level of a client in a long-term care facility. What question does the nurse ask to get this information? A. "Are your feet or hands cold-even when you are in bed?" B. "Do you feel more tired after you get up and go to the bathroom?" C. "How much exercise do you get?" D. "What is your endurance level?"

"Do you feel more tired after you get up and go to the bathroom?"

A nurse is assessing a client for hematologic function risks. The nurse seeks to determine whether there is a risk that cannot be reduced or eliminated. Which clinical health history question does the nurse ask to obtain this information? A. "Do you seem to have excessive bleeding or bruising?" B. "Does anyone in your family bleed a lot?" C. "Tell me what you eat in a day." D. "Where do you work?"

"Does anyone in your family bleed a lot?"

The nurse is teaching the client precautions to take while on warfarin (Coumadin) therapy. Which statement made by the client demonstrates that teaching has been effective? A. "I can use an electric razor or a regular razor." B. "Eating foods like green beans won't interfere with my Coumadin therapy." C. "If I notice I am bleeding a lot, I should stop taking Coumadin right away." D. "When taking Coumadin, I may notice some blood in my urine."

"Eating foods like green beans won't interfere with my Coumadin therapy."

A 52-year-old client relates to the nurse that she has never had a mammogram because she is terrified that she will have cancer. Which response by the nurse is therapeutic? A. "Don't worry, most lumps are discovered by women during breast self-examination." B. "Does anyone in your family have breast cancer?" C. "Finding a cancer in the early stages increases the chance for cure." D. "Have you noticed a lump or thickening in your breast?"

"Finding a cancer in the early stages increases the chance for cure."

The nurse is teaching postmastectomy exercises to the client. Which statement made by the client indicates that teaching has been effective? A. "For the pulley exercise, I'll drape a 6-foot-long rope over a shower curtain rod or over the top of a door." B. "In rope turning, I'll hold the rope with my arms flexed." C. "In rope turning, I'll start by making large circles." D. "With hand wall climbing, I'll walk my hands up the wall and back down until they are at waist level."

"For the pulley exercise, I'll drape a 6-foot-long rope over a shower curtain rod or over the top of a door."

A nurse is educating a group of young women who have sickle cell disease (SCD). Which comment from a class member requires correction? A. "Frequent handwashing is an important habit for me to develop." B. "Getting an annual 'flu shot' would be dangerous for me." C. "I must take my penicillin pills as prescribed-all the time." D. "The pneumonia vaccine is protection that I need."

"Getting an annual 'flu shot' would be dangerous for me."

A nurse is assessing an adult client's endurance in performing ADLs. What question does the nurse ask the client? A. "Can you prepare your own meals?" B. "Has your weight changed by 5 pounds or more this year?" C. "How is your energy level-compared with last year?" D. "What medications do you take daily, weekly, monthly?"

"How is your energy level-compared with last year?"

A nurse is teaching a hospitalized client who is being discharged about how to care for a peripherally inserted central catheter (PICC) line. Which client statement indicates a need for further education? A. "I can continue my 20-mile running schedule as I have for the past 10 years." B. "I can still go about my normal activities of daily living." C. "I have less chance of getting an infection because the line is not in my hand." D. "The PICC line can stay in for months."

"I can continue my 20-mile running schedule as I have for the past 10 years."

Which statement by the client with a recent cardiovascular diagnosis indicates maladaptive denial? A. "I don't know how I am going to change my lifestyle." B. "I don't need to change. It hasn't killed me yet." C. "I don't think it is as bad as the doctors say." D. "I will have to change my diet and exercise more."

"I don't need to change. It hasn't killed me yet."

When caring for a client with an abdominal aortic aneurysm (AAA), the nurse suspects dissection of the aneurysm when the client states which of these? A. "I feel my heart beating in my abdominal area." B. "I just started to feel a tearing pain in my belly." C. "I have a headache. May I have some acetaminophen?" D. "I have had hoarseness for a few weeks."

"I just started to feel a tearing pain in my belly."

The nurse is giving a group presentation on cancer prevention and recognition. Which statement by an older adult client indicates understanding of the nurse's instructions? A. "Cigarette smoking always causes lung cancer." B. "Taking multivitamins will prevent me from developing cancer." C. "If I have only one shot of whiskey a day, I probably will not develop cancer." D. "I need to report the pain going down my legs to my health care provider."

"I need to report the pain going down my legs to my health care provider."

Which client statement indicates that stem cell transplantation that is scheduled to take place in his home is not a viable option? A. "I don't feel strong enough, but my wife said she would help." B. "I was a nurse, so I can take care of myself." C. "I will have lots of medicine to take." D. "We live 5 miles from the hospital."

"I was a nurse, so I can take care of myself."

Which statement made by the client allows the nurse to recognize whether the client who is receiving brachytherapy for ovarian cancer understands the treatment plan? A. "I may lose my hair during this treatment." B. "I must be positioned in the same way during each treatment." C. "I will have a radioactive device in my body for a short time." D. "I will be placed in a semiprivate room for company."

"I will have a radioactive device in my body for a short time."

The nurse is teaching the young female client how to prevent venous thromboembolism specific to her hospital stay after intensive orthopedic surgery. Which statement made by the client indicates the need for further teaching? A. "I must stop taking my birth control pills." B. "I should drink lots of water so I don't get dehydrated." C. "I should exercise my legs when I have been sitting or standing for a long time." D. "If I wear pantyhose, I won't have to wear the stockings the hospital gives me."

"If I wear pantyhose, I won't have to wear the stockings the hospital gives me."

A client on anticoagulant therapy is being discharged. Which statement indicates that the client has a correct understanding of this therapy's purpose or action? A. "It is to dissolve blood clots." B. "It might cause me to get injured more often." C. "It should prevent my blood from clotting." D. "It will thin my blood."

"It should prevent my blood from clotting."

The client who has had a mastectomy asks the nurse about breast reconstructive surgery. Which statement by the nurse about breast reconstruction is true? A. "Many women want breast reconstruction using their own tissue immediately after mastectomy." B. "Placement of saline- or gel-filled prostheses is not recommended because of the nature of the surgery." C. "Reconstruction of the nipple-areola complex is the first stage in the reconstruction of the breast." D. "The surgeon should offer the option of breast reconstruction surgery once healing has occurred after the mastectomy."

"Many women want breast reconstruction using their own tissue immediately after mastectomy."

A nurse is inserting a peripheral intravenous (IV) catheter. Which client statement is of greatest concern during this procedure? A. "I hate having IVs started." B. "It hurts when you are inserting the line." C. "My hand tingles when you poke me." D. "My IV lines never last very long."

"My hand tingles when you poke me."

The client has just undergone arterial revascularization. Which statement by the client indicates a need for further teaching related to postoperative care? A. "My leg might turn very white after the surgery." B. "I should be concerned if my foot turns blue." C. "I should report a fever or any drainage." D. "Warmness, redness, and swelling are expected."

"My leg might turn very white after the surgery."

A client who is scheduled to undergo radiation for prostate cancer is admitted to the hospital by the registered nurse. Which statement by the client is most important to communicate to the physician? A. "I am allergic to iodine." B. "My urinary stream is very weak." C. "My legs are numb and weak." D. "I am incontinent when I cough."

"My legs are numb and weak."

The nurse is teaching the 47-year-old female client about recommended screening practices for breast cancer. Which statement by the client indicates understanding of the nurse's instructions? A. "My mother and grandmother had breast cancer, so I am at risk." B. "I get a mammography every 2 years since I turned 30." C. "A clinical breast examination is performed every month since I turned 40." D. "A CT scan will be done every year after I turn 50."

"My mother and grandmother had breast cancer, so I am at risk."

A nurse is teaching a group of unlicensed assistive personnel (UAP) about fluid intake principles for older adults. What does the nurse tell them? A. "Be careful not to overload them with too many oral fluids." B. "Offer fluids that they prefer frequently and on a regular schedule." C. "Restrict their fluids in the evening hours if they are incontinent." D. "Wake them every 2 hours during the night with a drink."

"Offer fluids that they prefer frequently and on a regular schedule."

A clinic nurse is discharging a 20-year-old client who had a bone marrow aspiration performed. What does the nurse advise the client to do? A. "Avoid contact sports or activity that may traumatize the site for 24 hours." B. "Inspect the site for bleeding every 4 to 6 hours." C. "Place an ice pack over the site to reduce the bruising." D. "Take a mild analgesic, such as 2 aspirin, for pain or discomfort at the site."

"Place an ice pack over the site to reduce the bruising."

The nurse is discussing treatment options with the client newly diagnosed with breast cancer. Which statement by the client indicates a need for further teaching? A. "Hormonal therapy is only used to prevent the growth of cancer. It won't get rid of it." B. "I might have chemotherapy before surgery." C. "If I get radiation, I am not radioactive to others." D. "Radiation will remove the cancer, so I might not need surgery."

"Radiation will remove the cancer, so I might not need surgery."

A client who has just been notified that the breast biopsy indicates a malignancy tells the nurse, "I just don't know how this could have happened to me." Which of these responses by the nurse will be most appropriate? A. "Tell me what you mean when you say you don't know how this could have happened to you." B. "Do you have a family history that might make you more likely to develop breast cancer?" C. "Would you like me to help you find more information about how breast cancer develops?" D. "Many risk factors for breast cancer have been identified, so it is difficult to determine what might have caused it."

"Tell me what you mean when you say you don't know how this could have happened to you."

The nurse reviews the chart of the client admitted with a diagnosis of glioblastoma with a T1NXM0 classification. Which explanation does the nurse offer when the client asks what the terminology means? A. "Two lymph nodes are involved in this tumor of the glial cells, and another tumor is present." B. "The brain tumor measures about 1 to 2 cm and shows no regional lymph nodes and no distant metastasis." C. "This type of tumor in the brain is small with some lymph node involvement; another tumor is present somewhere else in your body." D. "Glioma means this tumor is benign, so I will have to ask your health care provider the reason for the chemotherapy and radiation."

"The brain tumor measures about 1 to 2 cm and shows no regional lymph nodes and no distant metastasis."

A client with a low platelet count asks why platelets are important. How does the nurse answer? A. "Platelets make blood clots for you." B. "Blood clotting is prevented by your platelets." C. "The clotting process begins with your platelets." D. "Your platelets finish the clotting process."

"The clotting process begins with your platelets."

A client asks the nurse about early detection of breast masses. Which statement by the nurse about early detection of breast masses is correct? A. "A yearly breast examination by a health care provider can substitute for breast self-examination (BSE)." B. "Detection of breast cancer before axillary node invasion yields the same survival rate." C. "Mammography as a baseline screening is recommended by the American Cancer Society at 30 years of age." D. "The goal of screening for breast cancer is early detection because BSE does not prevent breast cancer."

"The goal of screening for breast cancer is early detection because BSE does not prevent breast cancer."

A client with leukemia is being discharged from the hospital. After hearing a nurse's instructions to keep regularly scheduled follow-up provider appointments, the client says, "I don't have transportation." How does the nurse respond? A. "A pharmaceutical company might be able to help." B. "I might be able to take you." C. "The local American Cancer Society may be able to help." D. "You can take the bus."

"The local American Cancer Society may be able to help."

A client with anemia asks the nurse, "Do most people have the same number of red blood cells?" How does the nurse respond? A. "No, they don't." B. "The number varies with gender, age, and general health." C. "Yes, they do." D. "You have fewer red blood cells because you have anemia."'

"The number varies with gender, age, and general health."

A nurse is reinforcing information about genetic counseling to a client with sickle cell disease (SCD) who has a healthy spouse. What information does the nurse include? A. "Sickle cell disease will be inherited by your children." B. "The sickle cell trait will be inherited by your children." C. "Your children will have the disease, but your grandchildren will not." D. "Your children will not have the disease, but your grandchildren could."

"The sickle cell trait will be inherited by your children."

A client who used to work as a nurse asks, "Why is the hospital using a 'fancy new IV' without a needle? That seems expensive." How does the nurse respond? A. "OSHA, the government, requires us to use this new type of IV." B. "These systems are designed to save time, not money." C. "They minimize health care workers' exposures to contaminated needles." D. "They minimize your exposure to contaminated needles."

"They minimize health care workers' exposures to contaminated needles."

A client with thrombocytopenia is being discharged. What information does the nurse incorporate into the teaching plan for this client? A. "Avoid large crowds." B. "Drink at least 2 liters of fluid per day." C. "Elevate your lower extremities when sitting." D. "Use a soft-bristled toothbrush."

"Use a soft-bristled toothbrush."

A client is being monitored for daily weights. The night nurse asks the nursing assistant for the morning weight, and the assistant replies, "She was sleeping so well, I didn't want to wake her to get her weight." How does the nurse respond? A. "Fast thinking! She really needs to rest after the night she had." B. "Get the information now, or I'll report you for not doing your job." C. "Never mind-I will do it myself." D. "Weigh her now. We need her weight daily, at the same time."

"Weigh her now. We need her weight daily, at the same time."

A nurse is instructing a client who is being discharged with a diagnosis of congestive heart failure (CHF). Which client statement indicates a correct understanding of CHF? A. "I can gain 2 pounds of water a day without risk." B. "I should call my provider if I gain more than 1 pound a week." C. "Weighing myself daily can determine if my caloric intake is adequate is effective." D. "Weighing myself daily can reveal increased fluid retention."

"Weighing myself daily can reveal increased fluid retention."

A client with multiple myeloma reports bone pain that is unrelieved by analgesics. How does the nurse respond to this client's problem? A. "Ask your doctor to prescribe more medication." B. "It is too soon for additional medication to be given." C. "I'll turn on some soothing classical music for you." D. "Would you like to try some relaxation techniques?"

"Would you like to try some relaxation techniques?"

A nurse is teaching a client about what to expect during a bone marrow biopsy. Which statement by the nurse accurately describes the procedure? A. "The doctor will place a small needle in your back and will withdraw some fluid." B. "You may experience a crunching sound or a scraping sensation as the needle punctures your bone." C. "You will be alone because the procedure is a sterile one; we cannot allow additional people to contaminate the area." D. "You will be sedated, so you will not be aware of anything."

"You may experience a crunching sound or a scraping sensation as the needle punctures your bone."

A client with anemia asks, "Why am I feeling tired all the time?" How does the nurse respond? A. "How many hours are you sleeping at night?" B. "You are not getting enough iron." C. "You need to rest more when you are sick." D. "Your cells are delivering less oxygen than you need."

"Your cells are delivering less oxygen than you need."

The client undergoing femoral popliteal bypass states that he is fearful he will lose the limb in the near future. Which response by the nurse is most therapeutic? A. "Are you afraid you will not be able to work?" B. "If you control your diabetes, you can avoid amputation." C. "Your concerns are valid; we can review some steps to limit disease progression." D. "What about the situation concerns you most?"

"Your concerns are valid; we can review some steps to limit disease progression."

After reviewing the laboratory test results, the nurse calls the health care provider about which client? A. 44-year-old receiving warfarin (Coumadin) with an international normalized ratio (INR) of 3.0 B. 46-year-old with a fever and a white blood cell (WBC) count of 500/µl C. 49-year-old with hemophilia and a platelet count of 150,000/mm3 D. 52-year-old who has had a hemorrhage with a reticulocyte count of 0.8%

46-year-old with a fever and a white blood cell (WBC) count of 500/µl

Which client does the medical unit charge nurse assign to an LPN/LVN? A. 23-year-old scheduled for a bone marrow biopsy with conscious sedation B. 35-year-old with a history of a splenectomy and a temperature of 100.9° F (38.3° C) C. 48-year-old with chronic microcytic anemia associated with alcohol use D. 62-year-old man with atrial fibrillation and an international normalized ratio (INR) of 6.6

48-year-old with chronic microcytic anemia associated with alcohol use

A 68-year-old man is admitted to the hospital with dehydration. He has a history of atrial fibrillation, congestive heart failure (CHF), and hypertension. His current medications are digoxin (Lanoxin), chlorothiazide (Diuril), and oral potassium supplements. He tells the nurse that he has had flu-like symptoms for the past week and has been unable to drink for the past 48 hours. The physician requests laboratory specimens to be drawn and an isotonic IV to be started. Which IV fluid does the nurse administer? A. 0.45% saline B. 5% dextrose in 0.45% saline C. 5% dextrose in Ringer's lactate D. 5% dextrose in water (D5W)

5% dextrose in water (D5W)

Normal range for WBC

5,000 - 10,000/mm3

Which client is at greatest risk for hypernatremia? A. 17 year-old with a serum blood glucose of 189 mg/dL B. 30-year-old on a low-salt diet C. 42-year-old receiving hypotonic fluids D. 54-year old who is sweating profusely

54-year old who is sweating profusely

A nurse is starting the shift by making rounds. Which client does the nurse decide to assess first? A. 42-year-old with anemia who is reporting shortness of breath when ambulating down the hallway B. 47-year-old who recently had a Rumpel-Leede test and is requesting a nurse to "look at the bruises on my arm" C. 52-year-old who has just had a bone marrow aspiration and is requesting pain medication D. 59-year old who has a nosebleed and is receiving heparin to treat a pulmonary embolism

59-year old who has a nosebleed and is receiving heparin to treat a pulmonary embolism

The nurse manager of the medical-surgical unit assigns which client to the LPN/LVN? A. 44-year-old admitted with dehydration who has a heart rate of 126 B. 54-year-old just admitted with hyperkalemia who takes a potassium-sparing diuretic at home C. 64-year-old admitted yesterday with heart failure who still has dependent pedal edema D. 74-year-old who has just been admitted with severe nausea, vomiting, and diarrhea

64-year-old admitted yesterday with heart failure who still has dependent pedal edema

The normal pH of the body's extracellular fluids (including blood)

7.35 to 7.45

The charge nurse on a medical-surgical unit is completing assignments for the day shift. Which client is assigned to the LPN/LVN? A. 44-year-old with congestive heart failure (CHF) who has gained 3 pounds since the previous day B. 58-year-old with chronic renal failure (CRF) who has a serum potassium level of 6 mEq/L C. 76-year-old with poor skin turgor who has a serum osmolarity of 300 mOsm/L D. 80-year-old with 3+ peripheral edema who has crackles throughout the posterior chest

76-year-old with poor skin turgor who has a serum osmolarity of 300 mOsm/L

Normal range for PaO2

80-100

33. Which client does the RN assess first? A. 32-year-old with pernicious anemia who needs a vitamin B12 injection B. 40-year-old with iron deficiency anemia who needs a Z-track iron injection C. 67-year-old with acute myelocytic leukemia with petechiae on both legs D. 81-year-old with thrombocytopenia and an increase in abdominal girth

81-year-old with thrombocytopenia and an increase in abdominal girth

Calcium (Ca2+) blood serum range

9.0-10.5 mg/dL

Chloride (Cl−) blood serum range

98-106 mEq/L

A newly graduated RN has just finished a 6-week orientation to the oncology unit. Which of these clients would be most appropriate to assign to the new graduate? A. A 30-year-old with acute lymphocytic leukemia who will receive combination chemotherapy today B. A 40-year-old with chemotherapy-induced nausea and vomiting who has had no urine output for 16 hours C. A 45-year-old with pancytopenia who will require IV administration of erythropoietin (Procrit) D. A 72-year-old with tumor lysis syndrome who is receiving normal saline IV at a rate of 250 mL/hr

A 45-year-old with pancytopenia who will require IV administration of erythropoietin (Procrit)

intermittent claudication

A characteristic leg pain experienced by patients with chronic peripheral arterial disease. Typically, patients can walk only a certain distance before a cramping muscle pain forces them to stop. As the disease progresses, the patient can walk only shorter and shorter distances before pain recurs. Ultimately, pain may occur even at rest.

An RN and an LPN/LVN, both of whom have several years of experience in the intensive care unit, are caring for a group of clients. Which client will be appropriate for the RN to assign to the LPN/LVN? A. A client with pulmonary edema who requires hourly monitoring of pulmonary artery wedge pressures B. A client who was admitted with peripheral vascular disease and needs assessment of the ankle-brachial index C. A client who has intermittent chest pain and requires teaching about myocardial nuclear perfusion imaging D. A client with acute coronary syndrome who has just been admitted and needs an admission assessment

A client who was admitted with peripheral vascular disease and needs assessment of the ankle-brachial index

Which client being cared for on the medical-surgical unit will be best to assign to a nurse who has floated from the intensive care unit (ICU)? A. Recent radical mastectomy client who requires chemotherapy administration B. Modified radical mastectomy client who needs discharge teaching C. Stage III breast cancer client who is requesting information about radiation and chemotherapy D. A client with a Jackson-Pratt drain in place who has just arrived from the postanesthesia care unit (PACU) after a quadrantectomy

A client with a Jackson-Pratt drain in place who has just arrived from the postanesthesia care unit (PACU) after a quadrantectomy

The RN working on an oncology unit has just received report on these clients. Which client should be assessed first? A. A client with chemotherapy-induced neutropenia who has just been admitted with an elevated temperature B. A client with lymphoma who will need administration of an antiemetic before receiving chemotherapy C. A client with metastatic breast cancer who is scheduled for external beam radiation in 1 hour D. A client with xerostomia associated with laryngeal cancer who needs oral care before breakfast

A client with chemotherapy-induced neutropenia who has just been admitted with an elevated temperature

Which of these clients would be appropriate to assign to the new nurse working on the unit? A. A client with diabetic ketoacidosis and change in mental status who has a pH of 7.18 B. A client with emphysema and cellulitis with a PaCO2 level of 58 mm Hg C. A client with reactive airway disease, wheezing, and a PaO2 level of 62 mm Hg D. A client with a small bowel obstruction and vomiting with a bicarbonate level of 40 mEq/L

A client with emphysema and cellulitis with a PaCO2 level of 58 mm Hg

Which client has pain most consistent with myocardial infarction (MI) requiring notification of the health care provider? A. A client with abdominal pain and belching B. A client with pressure in the mid abdomen and profound diaphoresis C. A client with dyspnea on exertion (DOE) and inability to sleep flat who sleeps on four pillows D. A client with claudication and fatigue

A client with pressure in the mid abdomen and profound diaphoresis

metabolic syndrome

A collection of related health problems with insulin resistance as a main feature. Other features include obesity, low levels of physical activity, hypertension, high blood levels of cholesterol, and elevated triglyceride levels. Metabolic syndrome increases the risk for coronary heart disease. Also called "syndrome X."

The home health RN is caring for a client who has a history of a kidney transplant and takes cyclosporine (Sandimmune) and prednisone (Deltasone) to prevent rejection. Which assessment data would be most important to communicate to the transplant team? A. The temperature is 96.6° F. B. The client reports joint pain. C. The oral mucosa appears pink and dry. D. A lump is palpable in the client's axilla.

A lump is palpable in the client's axilla.

A nurse is assessing a newly admitted client with thrombocytopenia. Which factor needs immediate intervention? A. A nosebleed B. Reports of pain C. Decreased urine output D. Increased temperature

A nosebleed

arterial ulcers

A painful complication in the patient with peripheral arterial disease. Typically, the ulcer is small and round, with a "punched out" appearance and well-defined borders. Ulcers develop on the toes (often the great toe), between the toes, or on the upper aspect of the foot. With prolonged occlusion, the toes can become gangrenous.

ankle-brachial index (ABI)

A ratio derived by dividing the ankle blood pressure by the brachial blood pressure; this calculation is used to assess the vascular status of the lower extremities. To obtain the ABI, a blood pressure cuff is applied to the lower extremities just above the malleoli. The systolic pressure is measured by Doppler ultrasound at both the dorsalis pedis and posterior tibial pulses. The higher of these two pressures is then divided by the higher of the two brachial pulses.

What is an INFUSATE?

A solution that is infused into the body

venous thromboembolism (VTE)

A term that refers to both deep vein thrombosis and pulmonary embolism; obstruction by a thrombus.

When assessing an older patient's hydration status, the nurse expects which assessment finding to be the best indicator of fluid losses or gains? A. Pretibial edema B. A weight gain of 2 pounds in one day C. Poor skin turgor D. The patient's report of feeling thirsty

A weight gain of 2 pounds in one day

The nurse caring for a client who has been NPO for 5 days and receiving only dextrose 5% in lactated Ringer's solution (4 liters daily) reviews the client's most recent arterial blood gas results and observes that the pH is now 7.28. What is the most likely explanation for this finding? A. Acidosis in response to the presence of excessive ketoacids B. Acidosis in response to the presence of excessive lactic acid C. Alkalosis in response to the excessive loss of carbonic acid D. Alkalosis in response to the excessive loss of sulfuric acid

Acidosis in response to the presence of excessive ketoacids

A 68-year-old man is admitted to the hospital with dehydration. He has a history of atrial fibrillation, congestive heart failure (CHF), and hypertension. His current medications are digoxin (Lanoxin), chlorothiazide (Diuril), and potassium supplements. He tells a nurse that he has had flu-like symptoms for the past week and has been unable to drink for the past 48 hours. The nurse starts the client's IV and receives laboratory results, which include a potassium level of 2.7 mEq/L. The physician orders an IV potassium supplement. How does the nurse administer this medication? A. Added to an IV, not to exceed 20 mEq/hr B. Added to an IV, not to exceed 30 mEq/hr C. Rapid IV push, a 25-mEq dose D. Slow IV push, a 30-mEq dose

Added to an IV, not to exceed 20 mEq/hr

The nurse is caring for a client with dark-colored toe ulcers and blood pressure of 190/100. Which of these nursing actions should you delegate to the LPN/LVN? A. Assess leg ulcers for evidence of infection. B. Administer a clonidine patch for hypertension. C. Obtain a request from the doctor for a dietary consult. D. Develop a plan for discharge, and assess home care needs.

Administer a clonidine patch for hypertension.

What is the primary risk factor for breast cancer?

Advanced age

A 72-year-old client recovering from lung cancer surgery asks the nurse to explain how she developed cancer when she has never smoked. Which factor may explain the possible cause? A. A diagnosis of diabetes treated with insulin and diet B. An exercise regimen of jogging 3 miles four times a week C. A history of cardiac disease D. Advancing age

Advancing age

What determines the number of RBCs a person has?

Age Gender General Health

A client who is undergoing chemotherapy for breast cancer reports problems with concentration and memory. Which intervention is indicated at this time? A. Explain that this occurs in some clients and is usually permanent. B. Encourage the client that a small glass of wine may help her relax. C. Protect the client from infection. D. Allow the client an opportunity to express her feelings.

Allow the client an opportunity to express her feelings

ALTERED GI STRUCTURE AND FUNCTION

Altered gastrointestinal structure and function may occur, interfering with normal nutrition and healing.

Which statement about diagnostic cardiovascular testing is true? A. Complications of coronary arteriography include stroke, nonlethal dysrhythmias, arterial bleeding, and thromboembolism. B. An alternative to injecting a medium into the coronary arteries is intravascular ultrasonography. C. Halter monitoring allows periodic recording of cardiac activity during an extended period of time. D. The left side of the heart is catheterized first and may be the only side examined.

An alternative to injecting a medium into the coronary arteries is intravascular ultrasonography.

Fibrinogen

An iactive protein that is activated to from fibrin

3 signs of digoxin toxicity

Anorexia Nausea Visual disturbances

A client is being admitted to the burn unit from another hospital. The client has an intraosseous IV that was started 2 days ago, according to the client's medical record. What does the admitting nurse do first? A. Anticipates an order to discontinue the intraosseous IV and start an epidural IV B. Calls the previous hospital to verify the date C. Immediately discontinues the intraosseous IV D. Nothing; this is a long-term treatment

Anticipates an order to discontinue the intraosseous IV and start an epidural IV

A newly admitted client has an elevated reticulocyte count. Which disorder does the nurse suspect in this client? A. Aplastic anemia B. Hemolytic anemia C. Infectious process D. Leukemia

Aplastic anemia

A nurse is starting a peripheral IV catheter on a recently admitted client. What actions does the nurse perform before insertion of the line? Select all that apply. A. Applies povidone-iodine to clean skin, dries for 2 minutes B. Cleans the skin around the site C. Prepares the skin with 70% alcohol or chlorhexidine D. Shaves the hair around the area of insertion E. Wears clean gloves and touches the site only with fingertips after applying antiseptics

Applies povidone-iodine to clean skin, dries for 2 minutes Cleans the skin around the site Prepares the skin with 70% alcohol or chlorhexidine

When caring for a client with a pulse oximetry level of 89%, which action should the nurse take first? A. Get the client out of bed. B. Apply oxygen as prescribed. C. Notify the client's physician. D. Auscultate breath sounds.

Apply oxygen as prescribed.

The nurse is caring for a client with hypoxemia and metabolic acidosis. Which of these tasks can be delegated to the nursing assistant who is helping with the client's care? A. Assess the client's respiratory pattern. B. Increase the IV normal saline to 120 mL/hr. C. Titrate O2 to maintain an O2 saturation of 95% to 100%. D. Apply the pulse oximeter for continuous readings.

Apply the pulse oximeter for continuous readings.

A client has a bone marrow biopsy done. Which nursing intervention is the priority postprocedure? A. Applying pressure to the biopsy site B. Inspecting the site for ecchymoses C. Sending the biopsy specimens to the laboratory D. Teaching the client about avoiding vigorous activity

Applying pressure to the biopsy site

hyperventilation

As the amount of CO2 begins to rise above normal in brain blood and tissues, these central receptors trigger the neurons to increase the rate and depth of breathing

Lactose intolerance caused by a deficiency of the enzyme lactase occurs in 75% to 90% of all

Asians, Blacks, and American Indians

The nurse is caring for a group of clients with acidosis. The nurse recognizes that Kussmaul respirations are consistent with which situation? A. Client receiving mechanical ventilation B. Use of hydrochlorothiazide C. Aspirin overdose D. Administration of sodium bicarbonate

Aspirin overdose

The nurse has received in report that the client receiving chemotherapy has severe neutropenia. Which of the following does the nurse plan to implement? Select all that apply. A. Assess for fever. B. Observe for bleeding. C. Administer pegfilgrastim (Neulasta). D. Do not permit fresh flowers or plants in the room. E. Do not allow his 16-year-old son to visit. F. Teach the client to omit raw fruits and vegetables from his diet.

Assess for fever. Administer pegfilgrastim (Neulasta). Do not permit fresh flowers or plants in the room. Teach the client to omit raw fruits and vegetables from his diet.

The nurse observes that the hand grip of the client with hypophosphatemia has diminished in strength since the last assessment 2 hours ago. What is the nurse's primary intervention? A. Documents the finding as the only action B. Assesses respiratory status immediately C. Applies elastic wraps to the lower extremities D. Assesses deep tendon reflexes

Assesses respiratory status immediately

A nurse who is starting the shift finds a client with an IV that is leaking all over the bed linens. What does the nurse do initially? A. Assesses the insertion site B. Checks connections C. Checks the infusion rate D. Discontinues the IV and starts another

Assesses the insertion site

Which statement reflects correct cardiac physical assessment technique? A. Auscultate the aortic valve in the second intercostal space at the right sternal border. B. Evaluate for orthostatic hypotension by moving the client from a standing to a reclining position. C. Palpate the apical pulse over the third intercostal space in the midclavicular line. D. Assess for carotid bruit by auscultating over the anterior neck.

Auscultate the aortic valve in the second intercostal space at the right sternal border.

The nurse is conducting a community health education class on diet and cancer risk reduction. What should be included in the discussion? A. Limit sodium intake. B. Avoid beef and processed meats. C. Increase consumption of whole grains. D. Eat "colorful fruits and vegetables," including greens. E. Avoid gas-producing vegetables such as cabbage.

Avoid beef and processed meats. Increase consumption of whole grains. Eat "colorful fruits and vegetables," including greens.

The client is struggling with body image after breast cancer surgery. Which behavior indicates to the nurse that the client is maladaptive? A. Avoiding eye contact with staff B. Saying, "I feel like less of a woman" C. Requesting a temporary prosthesis immediately D. Saying, "This is the ugliest scar ever"

Avoiding eye contact with staff

What is the first renal control mechanism?

Bicarbonate reabsorption

What allows RBCs to change their shape without breaking as they pass through narrow, winding capillaries?

Biconcave disc shape Flexible membrane

Hypocalcemia can occur in which ethnic group because of lactose intolerance? A. Blacks B. Caucasians C. Inuits D. Hispanics

Blacks

When caring for a client with an 8-cm abdominal aortic aneurysm (AAA), the nurse recognizes that which problem must be addressed immediately to prevent rupture? A. Heart rate 52 B. Blood pressure 192/102 C. Report of constipation D. Anxiety

Blood pressure 192/102

What are the risk factors for the development of leukemia? Select all that apply. A. Bone marrow hypoplasia B. Chemical exposure C. Down syndrome D. Ionizing radiation E. Multiple blood transfusions F. Prematurity at birth

Bone marrow hypoplasia Chemical exposure Down syndrome Ionizing radiation

The client is a 56-year-old man admitted with a diagnosis of acute myelogenous leukemia (AML). He is prescribed IV cytosine arabinoside for 7 days and an infusion of daunorubicin for the first 3 days. What is the major side effect of this therapy? A. Bone marrow suppression B. Liver toxicity C. Nausea D. Stomatitis

Bone marrow suppression

Which assessment finding will alert the nurse to a worsening of the client's hyponatremia? A. The client is anxious and combative. B. Diastolic blood pressure has increased by 8 mm Hg. C. Bowel sounds are hyperactive in all abdominal quadrants. D. Deep tendon reflexes have changed from 1+ to 2+.

Bowel sounds are hyperactive in all abdominal quadrants.

The nurse receives report on a client with a glioblastoma. Recognizing that cancers are classified by their tissue of origin, the nurse begins to plan care for a client with which type of cancer? A. Liver B. Smooth muscle C. Fatty tissue D. Brain

Brain

The client has a diagnosis of lung cancer. To which areas does the nurse anticipate that this client's tumor may metastasize? Select all that apply. A. Brain B. Bone C. Lymph nodes D. Kidneys E. Liver

Brain Bone Lymph nodes Liver

What are the most common method of breast reconstruction used today?

Breast expanders

The nurse is teaching a client who is receiving an anti-estrogen drug about the side effects she may encounter. Which of these should the nurse include in the discussion? Select all that apply. A. Heavy menses B. Smooth facial skin C. Hyperkalemia D. Breast tenderness E. Weight loss F. Deep vein thrombosis (DVT)

Breast tenderness Deep vein thrombosis (DVT)

Which signs or symptoms should the nurse report immediately because they indicate thrombocytopenia secondary to cancer chemotherapy? Select all that apply. A. Bruises B. Fever C. Petechiae D. Epistaxis E. Pallor

Bruises Petechiae Epistaxis

The lungs control the amount of?

CO2 that is retained or exhaled.

The nurse manager of a medical-surgical unit is completing assignments for the day shift staff. The client with which electrolyte laboratory value is assigned to the LPN/LVN? A. Calcium level of 9.5 mg/dL B. Magnesium level of 4.1 mEq/L C. Potassium level of 6.0 mEq/L D. Sodium level of 120 mEq/L

Calcium level of 9.5 mg/dL

REDUCED OXYGENATION

Cancer can disrupt respiratory function and gas exchange in several ways and often results in death.

REDUCED IMMUNITY AND BLOOD-PRODUCING FUNCTIONS

Cancers can cause reduced immunity and blood-producing functions, resulting in different types of hematologic and immunologic abnormalities.

A client is admitted to the cardiothoracic surgical intensive care unit (ICU) after cardiac bypass surgery. The client is still sedated on a ventilator and has an arterial catheter in the right wrist. What assessment does the nurse make to ensure patency of the client's arterial line? A. Blood pressure B. Capillary refill and pulse C. Neurologic function D. Questions the client about the pain level at the site

Capillary refill and pulse

Improves pump function by increasing contractility and decreasing heart rate

Cardiac glycosides or inotropic agents such as digoxin

_______ changes are the most severe problems from hyperkalemia and are the most common cause of death in patients with hyperkalemia.

Cardiovascular

________ changes are the most serious and life-threatening problems of hypercalcemia.

Cardiovascular

hypoventilation

Central receptors also sense low CO2 levels and stop or slow the neuron activity in the respiratory centers, decreasing the rate and depth of breathing

A nurse is admitting clients to the same-day surgery unit. Which insertion site for routine peripheral venous catheters does the nurse choose most often? A. Back of the hand for an older adult B. Cephalic vein of the forearm C. Lower arm on the side of a radical mastectomy D. Subclavian vein

Cephalic vein of the forearm

________ are the most obvious problems of hyponatremia

Cerebral changes

Which of the following findings would alarm the nurse when caring for a client receiving chemotherapy who has a platelet count of 17,000/mm3? A. Increasing shortness of breath B. Diminished bilateral breath sounds C. Change in mental status D. Weight gain of 4 pounds in 1 day

Change in mental status

A client who is receiving intravenous antibiotic treatments every 6 hours has an intermittent IV set that was opened and begun 20 hours ago. What action does the nurse take? A. Changes the set immediately B. Changes the set in about 4 hours C. Changes the set in the next 12 to 24 hours D. Nothing; the set is for long-term use

Changes the set in about 4 hours

When caring for a client who has received recombinant tissue plasminogen activator (t-PA), for a large deep vein thrombosis (DVT), the nurse would be most concerned if the client developed which of these? A. Small amount of blood at the IV insertion site B. Heavy menstrual bleeding C. +1 pitting edema of the affected extremity D. Client stating that the year is 1967

Client stating that the year is 1967

Which client is most likely to exhibit the following ABG results: pH, 7.30; PaCO2, 49; HCO , 26; PO2, 76? A. Client with kidney failure B. Client taking hydromorphone (Dilaudid) C. Client with anxiety disorder D. Client with hyperkalemia

Client taking hydromorphone (Dilaudid)

The client is a 56-year-old man admitted with a diagnosis of acute myelogenous leukemia (AML). He is prescribed IV cytosine arabinoside for 7 days and an infusion of daunorubicin for the first 3 days. He develops an infection. What knowledge does the nurse use to determine that the appropriate antibiotic has been prescribed for this client? A. Evaluating the client's liver function tests (LFTs) and serum creatinine levels B. Evaluating the client's white blood cell (WBC) count level C. Checking the culture and sensitivity test results to be certain that the requested antibiotic is effective against the organism causing the infection D. Recognizing that vancomycin (Vancocin) is the drug of choice used to treat all infections in clients with AML

Checking the culture and sensitivity test results to be certain that the requested antibiotic is effective against the organism causing the infection

A nurse is administering a drug to a client through an implanted port. Before giving the medication, what does the nurse do to ensure safety? A. Administers 5 mL of a heparinized solution B. Checks for blood return C. Flushes the port with 10 mL of normal saline D. Palpates the port for stability

Checks for blood return

Adjuvant therapy

Chemotherapy used along with surgery or radiation.

The nurse is caring for a client with suspected pericarditis. Which signs and symptoms support this? Select all that apply. A. Squeezing, vise-like chest pain B. Chest pain relieved by sitting upright C. Chest and abdominal pain relieved by antacids D. Sudden-onset chest pain relieved by NSAIDs E. Pain in the chest described as sharp or stabbing

Chest pain relieved by sitting upright Sudden-onset chest pain relieved by NSAIDs Pain in the chest described as sharp or stabbing

inflow disease

Chronic peripheral arterial disease with obstruction at or above the common iliac artery, abdominal aorta, or profunda femoris artery. The patient experiences discomfort in the lower back, buttocks, or thighs after walking a certain distance. The pain usually subsides with rest.

Outflow disease

Chronic peripheral arterial disease with obstruction at or below the superficial femoral or popliteal artery. The patient experiences burning or cramping in the calves, ankles, feet, and toes after walking a certain distance; the pain usually subsides with rest.

Erythema at access site with or without pain

Class 1 Phlebitis

Pain at access site with erythema and/or oedema

Class 2 Phlebitis

Pain at access site with erythema and/or oedema Streak formation Palpable venous cord

Class 3 Phlebitis

Pain at access site with erythema and/or oedema Streak formation Palpable venous cord > 1 inch in length Purulent drainag

Class 4 Phlebitis

An RN is assessing a 70-year-old client admitted to the unit with severe dehydration. Which finding requires immediate intervention by the nurse? A. Client behavior that changes from anxious and restless to lethargic and confused B. Deep furrows on the surface of the tongue C. Poor skin turgor with tenting remaining for 2 minutes after the skin is pinched D. Urine output of 950 mL for the past 24 hours

Client behavior that changes from anxious and restless to lethargic and confused

Which client will be best for the charge nurse to assign to a new graduate RN who has completed 2 months of orientation to the coronary care unit? A. Client who has a new diagnosis of heart failure and needs a pulmonary artery catheter inserted B. Client who has just arrived after a coronary arteriogram and has vital signs requested every 15 minutes C. Client with acute electrocardiographic changes who is requesting nitroglycerin for left anterior chest pain D. Client who has many questions about the electrophysiology studies (EPS) scheduled for today

Client who has just arrived after a coronary arteriogram and has vital signs requested every 15 minutes

When caring for a group of clients at risk for respiratory acidosis, the nurse identifies which person as at highest risk? A. An athlete in training B. Pregnant woman with hyperemesis gravidarum C. Person with uncontrolled diabetes D. Client who smokes cigarettes

Client who smokes cigarettes

The registered nurse is teaching a nursing student about the importance of observing for bone marrow suppression during chemotherapy. Select the person who displays bone marrow suppression. A. Client with hemoglobin of 7.4 and hematocrit of 21.8 B. Client with diarrhea and potassium level of 2.9 mEq/L C. Client with 250,000 platelets D. Client with 5000 white blood cells/mm3

Client with hemoglobin of 7.4 and hematocrit of 21.8

A nurse is documenting peripheral venous catheter insertion for a client. What does the nurse include in the note? Select all that apply. A. Client's name and hospital number B. Client's response to the insertion C. Date and time inserted D. Type and size of device E. Type of dressing applied F. Vein that was used for insertion

Client's response to the insertion Date and time inserted Type and size of device Type of dressing applied Vein that was used for insertion

A 70-year-old female is admitted to the hospital with heart failure, shortness-of-breath (SOB), and 3+ pitting edema in her lower extremities. Her medications are furosemide (Lasix), digoxin (Lanoxin), and an angiotensin-converting enzyme (ACE) inhibitor (Lotensin). She states that she stopped taking her Lasix because she did not think that it was helping her heart failure. Her physician orders furosemide (Lasix) 5 mg IV push. Ten (10) hours after receiving the Lasix, the client's potassium (K+) level is 2.5 mEq/L. Knowing all of the client's medications, what problem(s) does the nurse anticipate in this client? A. Clinical manifestations of digoxin toxicity B. Increased heart rate and blood pressure (BP) C. Increased signs of congestive heart failure (CHF) D. Signs and symptoms of hypernatremia

Clinical manifestations of digoxin toxicity

The nurse suspects that the client has developed an acute arterial occlusion of the right lower extremity based on which of the following? Select all that apply. A. Hypertension B. Tachycardia C. Bounding right pedal pulses D. Cold right foot E. Numbness and tingling of right foot F. Mottling of right foot and lower leg

Cold right foot Numbness and tingling of right foot Mottling of right foot and lower leg

The nurse in a coronary care unit interprets information from hemodynamic monitoring. The client has a cardiac output of 2.4 L/min. Which of the following actions should be taken by the nurse? A. No intervention is needed; this is a normal reading. B. Collaborate with the physician to administer a positive inotropic agent. C. Administer a stat dose of metoprolol (Lopressor). D. Ask the client to perform the Valsalva maneuver.

Collaborate with the physician to administer a positive inotropic agent.

The client has been diagnosed with breast cancer. Which treatment option chosen by the client requires the nurse to discuss with the client the necessity of considering additional therapy? A. Chemotherapy B. Complementary and alternative therapy (CAM) C. Hormonal therapy D. Neoadjuvant therapy

Complementary and alternative therapy (CAM)

Which teaching is most appropriate for a client with chemotherapy-induced neuropathy? A. Bathe in cold water. B. Wear cotton gloves when cooking. C. Consume a diet high in fiber. D. Make sure shoes are snug.

Consume a diet high in fiber.

When administering furosemide (Lasix) to a client who states she does not like bananas or orange juice, the nurse recommends that the client try which intervention to maintain potassium levels? A. Increase red meat in the diet. B. Consume melons and baked potato. C. Add several portions of dairy products each day. D. Try replacing your usual breakfast with oatmeal or cream of wheat.

Consume melons and baked potato.

A nurse checking an IV fluid order questions its accuracy. What does the nurse do first? A. Asks the charge nurse about the order B. Contacts the health care provider who ordered it C. Contacts the pharmacy for clarification D. Starts the fluid as ordered, with plans to check it later

Contacts the health care provider who ordered it

The two major categories of lymphoma among adults are

Hodgkin's lymphoma and non-Hodgkin's lymphoma.

A nurse is teaching a client with vitamin B12 deficiency anemia about dietary intake. Which type of food does the nurse encourage the client to eat? A. Dairy products B. Grains C. Leafy vegetables D. Starchy vegetables

Dairy products

As adults age, which common physiologic change is likely to alter their hydration status? A. Adrenal gland growth B. Decreased muscle mass C. Increased thirst mechanism D. Poor skin turgor

Decreased muscle mass

Signs of left sided heart failure

Decreasing BP Anxious, pale, tachycardic Crackles, wheezes, dyspnea, and cough Pulmonary edema

WHat are some age related changes in the heart

Decreasing contractility Decresed elasticity (Stiffens valves) Decreased SA node cells Thickened valves

When taking the blood pressure of a very ill client, the nurse observes that the client's hand undergoes flexion contractions. What is the nurse's primary intervention? A. Placing the client in a high Fowler's position and administering intravenous fluids B. Deflating the blood pressure cuff and administer oxygen C. Documenting the finding as the only action D. Notifying the health care provider

Deflating the blood pressure cuff and administer oxygen

signs of right heart failure

Dependent edema increased central venous pressure jugular vein distention abdominal engorgement decreased urinary output

hyperosmotic or hypertonic

Describes fluids with osmolarities (solute concentrations) greater than 300 mOsm/L; have a greater osmotic pressure than do isosmotic fluids and tend to pull water from the isosmotic fluid space into the fluid space until an osmotic balance occurs.

hypo-osmotic or hypotonic

Describes fluids with osmolarities of less than 270 mOsm/L. Have a lower osmotic pressure than isosmotic fluids, and water tends to be pulled from the c fluid space into the isosmotic fluid space until an osmotic balance occurs.

Monocyte

Destruction of bacteria and cellular debris Mature into macrophage

Which nursing action does the RN delegate to unlicensed assistive personnel (UAP) who are assisting with the care of a female client with anemia? A. Asking the client about the amount of blood loss with each menstrual period B. Checking for sternal tenderness while applying fingertip pressure C. Determining the respiratory rate before and after the client walks 20 feet D. Monitoring her oral mucosa for pallor, bleeding, or ulceration

Determining the respiratory rate before and after the client walks 20 feet

Which situation can cause a client to experience "insensible water loss"? Select all that apply. A. Diarrhea B. Dry, hot weather C. Fever D. Increased respiratory rate E. Nausea F. Mechanical ventilation

Diarrhea Dry, hot weather Fever Increased respiratory rate Mechanical ventilation

Decreases circulating fluid and decreases preload

Diuretics

: A 77-year-old woman is brought to the emergency department by her family after she has had diarrhea for 3 days. The family tells a nurse that she has not been eating or drinking well, but that she has been taking her diuretics for congestive heart failure (CHF). Her laboratory results include a potassium level of 7.0 mEq/L. What does the nurse include in the client's medication teaching? Select all that apply. A. Daily weights are a poor indicator of fluid loss or gain. B. Diuretics can lead to fluid and electrolyte imbalances. C. Diuretics increase fluid retention. D. Laxatives can lead to fluid imbalance.

Diuretics can lead to fluid and electrolyte imbalances. Laxatives can lead to fluid imbalance.

The nurse presents a cancer prevention program to teens. Which of the following will have the greatest impact in cancer prevention? A. Avoid asbestos. B. Wear sunscreen. C. Get the human papilloma virus (HPV) vaccine. D. Do not smoke cigarettes.

Do not smoke cigarettes.

The nurse is caring for a client who is being treated for hypertensive emergency. Which medication prescribed for the client should the nurse question? A. Enalapril (Vasotec) B. Sodium nitroprusside (Nipride) C. Dopamine (Intropin) D. Clevidipine (butyrate)

Dopamine (Intropin)

The nurse is assessing a client with lung cancer. Which symptom does the nurse anticipate finding? A. Easy bruising B. Dyspnea C. Night sweats D. Chest wound

Dyspnea

A nurse is assessing a client with anemia. Which clinical manifestations does the nurse expect to see? Select all that apply. A. Decreased breath sounds B. Dyspnea on exertion C. Elevated temperature D. Fatigue E. Pallor F. Tachycardia

Dyspnea on exertion Fatigue Pallor Tachycardia

The nurse is instructing a client with breast cancer who will be undergoing chemotherapy about side effects of doxorubicin (Adriamycin). Which side effect will the nurse instruct the client to report to the physician? A. Diaphoresis B. Dysphagia C. Edema D. Hearing loss

Edema

Which manifestation of an oncologic emergency requires the nurse to contact the health care provider immediately? A. New onset of fatigue B. Edema of arms and hands C. Dry cough D. Weight gain

Edema of arms and hands

The nurse is educating a group of clients about risk factors for cardiovascular disease. Which of these risk factors should be included in the discussion? A. Consuming a diet rich in fiber B. Elevated C-reactive protein levels C. Low blood pressure D. Elevated high-density lipoprotein (HDL) cholesterol level E. Smoking

Elevated C-reactive protein levels Smoking

hypertriglyceridemia

Elevated levels (150 mg/dL or above) of triglyceride in the blood.

A client develops fluid overload while in the intensive care unit. Which nursing intervention does the nurse perform first? A. Draws blood for laboratory tests B. Elevates the head of the bed C. Places the extremities in a dependent position D. Puts the client in a side-lying position

Elevates the head of the bed

When caring for a client who has had a colostomy created as part of a regimen to treat colon cancer, which activities would help to support the client in accepting changes in appearance or function? Select all that apply. A. Explain to the client that the colostomy is only temporary. B. Encourage the client to participate in changing the ostomy. C. Obtain a psychiatric consultation. D. Offer to have a person who is coping with a colostomy visit. E. Encourage the client and family members to express their feelings and concerns.

Encourage the client to participate in changing the ostomy. Offer to have a person who is coping with a colostomy visit. Encourage the client and family members to express their feelings and concerns.

Which action should the nurse take first for the client who is admitted to the emergency department (ED) with a panic attack and whose blood gases indicate respiratory alkalosis? A. Encourage the client to take slow breaths. B. Obtain a prescription for a fluid and electrolyte infusion. C. Administer oxygen using ED standard orders. D. Place an emergency cart close to the client's room.

Encourage the client to take slow breaths

A new nurse graduate is caring for a postoperative client with the following arterial blood gases (ABGs): pH, 7.30; PCO2, 60 mm Hg; PO2, 80 mm Hg; bicarbonate, 24 mEq/L; and O2 saturation, 96%. Which of these actions by the new graduate is indicated? A. Encourage the client to use the incentive spirometer and to cough. B. Administer oxygen by nasal cannula. C. Request a prescription for sodium bicarbonate from the health care provider. D. Inform the charge nurse that no changes in therapy are needed.

Encourage the client to use the incentive spirometer and to cough.

What intervention most effectively protects a client with thrombocytopenia? A. Avoiding the use of dentures B. Encouraging the use of an electric shaver C. Taking rectal temperatures D. Using warm compresses on trauma sites

Encouraging the use of an electric shaver

T-lymphcyte helper/Inducer T-cell

Enhances immune activity through the secretion of various factors, cytokines, and lymphokines

The nurse is instructing a client on how to perform breast self-examination (BSE). Which techniques will the nurse include in teaching the client about BSE? Select all that apply. A. Instruct the client to keep her arm by her side while performing the examination. B. Ensure that the setting in which BSE is demonstrated is private and comfortable. C. Ask the client to remove her shirt. The bra may be left in place. D. Ask the client to demonstrate her own method of BSE. E. Use the fingertips, which are more sensitive than the finger pads, to palpate the breasts.

Ensure that the setting in which BSE is demonstrated is private and comfortable. Ask the client to demonstrate her own method of BSE.

A nurse is to administer a unit of whole blood to a postoperative client. What does the nurse do to ensure the safety of the blood transfusion? A. Asks the client to both say and spell his or her full name before starting the blood transfusion B. Ensures that another qualified health care professional checks the unit before administering C. Checks the blood identification numbers with the laboratory technician at the Blood Bank at the time it is dispersed D. Makes certain that an IV solution of 0.9% normal saline is infusing into the client before starting the unit

Ensures that another qualified health care professional checks the unit before administering

What is an example of a growth factor that helps blood stem cells differentiate?

Erythropoietin (produced by the kidneys-help BSC turn into RBCs)

What is the priority nursing diagnosis for a patient with CHF?

Excessive fluid volume (Fluid imbalance)-- Must remove fluid before you are able to oxygenate

Etiology of Essential or Primary Hypertension

Family history of hypertension African-American ethnicity Hyperlipidemia Smoking Older than 60 years or postmenopausal Excessive sodium and caffeine intake Overweight/obesity Physical inactivity Excessive alcohol intake Low potassium, calcium, or magnesium intake Excessive and continuous stress

Which potential side effects should be included in the teaching plan for a client undergoing radiation therapy for laryngeal cancer? A. Fatigue B. Changes in color of hair C. Change in taste D. Changes in skin of the neck E. Difficulty swallowing

Fatigue Change in taste Changes in skin of the neck Difficulty swallowing

1st thing to do after catheter use

Flush catheter with saline or saline followed by heparinized saline failure to do this causes lumen occlusion Obtained in single dose containers or prefilled syringes

What is the second renal control mechanism?

Formation of acids

What is the third renal control mechanism?

Formation of ammonium

A nurse is caring for a client with sickle cell disease (SCD). Which action is most effective in reducing the potential for sepsis in this client? A. Administering prophylactic drug therapy B. Frequent and thorough handwashing C. Monitoring laboratory values to look for abnormalities D. Taking vital signs every 4 hours-day and night

Frequent and thorough handwashing

Types of rays produced from radiation

Gamma rays - most commonly used for radiation therapy because of their ability to deeply penetrate tissues. Beta particles - have much less ability to penetrate tissues and must be placed within or very close to the cancer cells to be effective (brachytherapy) Alpha particles - not used as part of radiation therapy

Common sites of metastasis for Melanoma

Gi tract, lymph nodes, lung, brain

The client is receiving chemotherapy treatment for breast cancer and asks for additional support for managing the associated nausea and vomiting. Which complementary therapy will the nurse suggest? A. Ginger B. Journaling C. Meditation D. Yoga

Ginger

Blood Stem Cells

Immature, specialized (undifferentiated) cells

A nurse is revising an agency's recommended central line catheter-related bloodstream infection prevention (CR-BSI) bundle. Which actions decrease the client's risk for this complication? Select all that apply. A. During insertion, draping the area around the site with a sterile barrier B. Immediately removing the client's venous access device (VAD) when it is no longer needed C. Making certain that observers of the insertion are instructed to look away during the procedure D. Thorough hand hygiene (i.e., no quick scrub) before insertion E. Using chlorhexidine for skin disinfection

Immediately removing the client's venous access device (VAD) when it is no longer needed Thorough hand hygiene (i.e., no quick scrub) before insertion Using chlorhexidine for skin disinfection

A 70-year-old female is admitted to the hospital with heart failure, shortness-of-breath (SOB), and 3+ pitting edema in her lower extremities. Her current medications are furosemide (Lasix), digoxin (Lanoxin), and an angiotensin-converting enzyme (ACE) inhibitor (Lotensin). She states that she stopped taking her Lasix because she did not think that it was helping her heart failure. Her physician orders furosemide (Lasix) 5 mg IV push. Which client assessment determines that the medication is working? A. Decreased blood pressure (BP) B. Increased heart rate C. Increased urine output D. Weight gain

Increased urine output

A client who has been newly diagnosed with leukemia is admitted to the hospital. Avoiding which potential problem takes priority in the client's nursing care plan? A. Fluid overload (overhydration) B. Hemorrhage C. Hypoxia D. Infection

Infection

The nurse explains to the client that which risk factor most likely contributed to his primary liver carcinoma? A. Infection with hepatitis B virus B. Consuming a diet high in animal fat C. Exposure to radon D. Familial polyposis

Infection with hepatitis B virus

A client has a low serum potassium level and is ordered a dose of parenteral potassium chloride (KCl). How does a nurse safely administer KCl to the client? A. Administers 5 mEq, intramuscularly (IM) B. Dilutes 200 mEq in 1 liter of normal saline and infuses at 100 mL/hr C. Infuses 10 mEq over a 1-hour period D. Pushes 5 mEq through a central access line

Infuses 10 mEq over a 1-hour period

A client is to receive an IV solution of 5% dextrose and half-normal saline at 125 mL/hr. Which system provides the safest method for the nurse to accurately administer this solution? A. Controller B. Glass container C. Infusion pump D. Syringe pump

Infusion pump

Which nursing intervention takes priority for a client admitted with severe metabolic acidosis? A. Perform medication reconciliation. B. Assess the client's strength in the extremities. C. Obtain a diet history for the past 3 days. D. Initiate cardiac monitoring.

Initiate cardiac monitoring.

A 77-year-old woman is brought to the emergency department by her family after she has had diarrhea for 3 days. The family tells the nurse that she has not been eating or drinking well, but that she has been taking her diuretics for congestive heart failure (CHF). Laboratory results include a potassium level of 7.0 mEq/L. Which medication(s) does the nurse anticipate administering? A. Insulin (regular insulin) and dextrose (D20W) B. Loperamide (Imodium) C. Sodium polystyrene sulfonate (Kayexalate) D. Supplemental potassium

Insulin (regular insulin) and dextrose (D20W)

The registered nurse is teaching a group of nursing students about malignant transformation. Which statement about the process of malignant transformation is true? A. Mutation of genes is an irreversible event that always leads to cancer development in the initiation phase. B. Insulin and estrogen enhance the division of an initiated cell during the promotion phase. C. Tumors form when carcinogens invade the gene structure of the cell in the latency phase. D. Nutrition of cancer cells is provided by tumor angiogenesis factor (TAF) in the promotion stage.

Insulin and estrogen enhance the division of an initiated cell during the promotion phase.

The nurse teaches the client that intraperitoneal chemotherapy will be delivered where? A. Into the veins of the legs B. Into the lung C. Into the heart D. Into the abdominal cavity

Into the abdominal cavity

A recently admitted client who is in sickle cell crisis requests "something for pain." What does the nurse administer? A. Intramuscular (IM) morphine sulfate B. Intravenous (IV) hydromorphone (Dilaudid) C. Oral ibuprofen (Motrin) D. Oral morphine sulfate (MS-Contin)

Intravenous (IV) hydromorphone (Dilaudid)

The nurse is caring for a client with an oxygen saturation of 88% and accessory muscle use. The nurse provides oxygen and anticipates which of these physician orders? A. Administration of IV sodium bicarbonate B. Computed tomography (CT) of the chest, stat C. Intubation and mechanical ventilation D. Administration of concentrated potassium chloride solution

Intubation and mechanical ventilation

What is a common anemia that can result from blood loss, poor GI absorption of iron, and an inadequate diet?

Iron deficiency anemia

What is the most common type of fluid loss problem?

Isotonic dehydration

What does a TransEsophogeal Echocardiogram (TEE) evaluate?

It evaluates the valves

A 56-year-old man is admitted with a diagnosis of acute myelogenous leukemia (AML). He is prescribed IV cytosine arabinoside for 7 days and an infusion of daunorubicin for the first 3 days. He is started on an antiviral agent. What are serious side effects of antiviral agents? Select all that apply. A. Cardiomyopathy B. Nephrotoxicity C. Ototoxicity D. Stroke

Nephrotoxicity Ototoxicity

What leukocytes are involved in inflammation?

Neutrophil Macrophage Monocyte Eosinophil Basophil

Which assessment finding indicates to the nurse that the client is at high risk for a malignant breast lesion? A. 1-cm freely mobile rubbery mass discovered by the client B. Ill-defined painful rubbery lump in the outer breast quadrant C. Backache and breast fungal infection D. Nipple discharge and dimpling

Nipple discharge and dimpling

WHat is the first drug given to patients with chest pain?

Nitroglycerin

Natural Killer Cell

Nonselectively attacks non-self cells, especially boy cells that have undergone mutation and become malignant Also attacks grafts and transplanted organs

Neutrophil

Nonspecific ingestion and phagocytosis of microorganisms and foreign protein

Macrophage

Nonspecific recognition of foreign proteins and microorganisms Ingestion and phagocytosis

WHat are the valves doing when you have a murmur?

Not closing effectively

What is used to "FLUSH" MIDLINE CATHS?

ONLY SALINE

Which action should the nurse delegate to experienced unlicensed assistive personnel (UAP) working in the cardiac catheterization laboratory? A. Assess preprocedure medications the client took that day. B. Have the client sign the consent form before the procedure is performed. C. Educate the client about the need to remain on bedrest after the procedure. D. Obtain client vital signs and a resting electrocardiogram (ECG).

Obtain client vital signs and a resting electrocardiogram (ECG).

An RN is caring for a client admitted with dehydration who requires a blood transfusion. Which nursing action does the RN delegate to unlicensed assistive personnel (UAP)? A. Inserting a small-gauge needle for intravenous (IV) access B. Evaluating a headache that develops during the transfusion C. Explaining to the client the purpose of the blood transfusion D. Obtaining baseline vital signs before blood administration

Obtaining baseline vital signs before blood administration

Which would be an appropriate task to delegate to unlicensed assistive personnel (UAP) working on a medical-surgical unit? A. Administering erythropoietin to a client with myelodysplastic syndrome B. Assessing skin integrity on an anemic client who fell during ambulation C. Assisting a client with folic acid deficiency in making diet choices D. Obtaining vital signs on a client receiving a blood transfusion

Obtaining vital signs on a client receiving a blood transfusion

A nurse is caring for a client with neutropenia who has a suspected infection. Which intervention does the nurse implement first? A. Hydrates the client with 1000 mL of IV normal saline B. Initiates the administration of prescribed antibiotics C. Obtains requested cultures D. Places the client on Bleeding Precautions

Obtains requested cultures

A nurse is planning care for a 72-year-old resident of a long-term care facility who has a history of dehydration. Which action does the nurse delegate to unlicensed assistive personnel (UAP)? A. Assessing oral mucosa for dryness B. Choosing appropriate oral fluids C. Monitoring skin turgor for tenting D. Offering fluids to drink every hour

Offering fluids to drink every hour

The nurse suspects that which client has the highest risk for breast cancer? A. Older adult woman with high breast density B. Nullipara older adult woman C. Obese older adult male with gynecomastia D. Middle-aged woman with high breast density

Older adult woman with high breast density

Which client does the charge nurse on a medical-surgical unit assign to the LPN/LVN? A. Cardiac client who has a diltiazem (Cardizem) IV infusion being titrated to maintain a heart rate between 60 and 80 B. Diabetic client admitted for hyperglycemia who is on an IV insulin drip and needs frequent glucose checks C. Older client admitted for confusion who has a heparin lock that needs to be flushed every 8 hours D. Postoperative client receiving blood products after excessive blood loss during surgery

Older client admitted for confusion who has a heparin lock that needs to be flushed every 8 hours

Which medication does the nurse plan to administer to a client before chemotherapy to decrease the incidence of nausea? A. Morphine B. Ondansetron (Zofran) C. Naloxone (Narcan) D. Diazepam (Valium)

Ondansetron (Zofran)

Which newly written physician prescription does the nurse administer first? A. Intravenous (IV) normal saline to a client with a serum sodium of 132 mEq/L B. Oral calcium supplements to a client with severe osteoporosis C. Oral phosphorus supplements to a client with acute hypophosphatemia D. Oral potassium chloride (KCl) to a client whose serum potassium is 3 mEq/L

Oral potassium chloride (KCl) to a client whose serum potassium is 3 mEq/L

A 25-year-old student has been taken to an urgent care clinic because of dehydration. She says she has had "the flu," with vomiting and diarrhea "all night" and admits that she has had very little to eat or drink. She now says the GI symptoms have subsided but she feels weak. The nurse expects which type of rehydration to occur? A. Oral rehydration therapy with water B. Oral rehydration therapy with a solution containing glucose and electrolytes C. Diuretic therapy D. IV fluid replacement

Oral rehydration therapy with a solution containing glucose and electrolytes

An older adult client is receiving furosemide (Lasix) for treatment of peripheral edema. Which nursing assessment data identify that the client is at risk for falling? A. Dry oral mucous membranes B. Orthostatic blood pressure changes C. Pulse rate of 72 beats/min and bounding D. Serum potassium level of 4.0 mEq/L

Orthostatic blood pressure changes

BlOOD TRANSFUSION are "accomplished" by use of what?

PACKED RED BLOOD CELLS

Those with acute arterial insufficiency often present with the "six P's" of ischemia:

Pain Pallor Pulselessness Paresthesia Paralysis Poikilothermy (coolness)

The client with peripheral arterial occlusive disease has undergone percutaneous transluminal angioplasty (PTA) of the lower extremity. What is essential for the nurse to assess after the procedure? A. Ankle-brachial index (ABI) B. Dye allergy C. Pedal pulses D. Gag reflex

Pedal pulses

A client recovering from a cardiac angiography develops slurred speech. What does the nurse do first? A. Maintains NPO (nothing by mouth) until this resolves B. Calls in another nurse for a second opinion C. Performs a complete neurologic assessment and notifies the physician D. Explains to the client and family that this is expected after sedation

Performs a complete neurologic assessment and notifies the physician

The nurse includes which of the following in teaching regarding the warning signs of cancer? Select all that apply. A. Persistent constipation B. Scab present for 6 months C. Curdlike vaginal discharge D. Axillary swelling E. Headache

Persistent constipation Scab present for 6 months Axillary swelling

An RN is caring for a client with end-stage liver disease that has resulted in ascites. Which action does the RN delegate to unlicensed assistive personnel (UAP)? A. Assessing skin integrity and abdominal distention B. Drawing blood from a central venous line for electrolyte studies C. Evaluating laboratory study results for the presence of hypokalemia D. Placing the client in a semi-Fowler's position

Placing the client in a semi-Fowler's position

A physician writes orders for a client who is admitted with a serum potassium (K) level of 6.9 mEq/L. What does the nurse implement first? A. Administering sodium polystyrene sulfonate (Kayexalate) orally. B. Ensuring that a potassium-restricted diet is ordered. C. Placing the client on a cardiac monitor. D. Teaching the client about foods that are high in potassium.

Placing the client on a cardiac monitor.

A nurse is reviewing complete blood count (CBC) data for a 76-year-old client. Which decreased value causes concern because it is not age related? A. Hemoglobin (Hgb) level B. Platelet (thrombocyte) count C. Red blood cell (RBC) count D. White blood cell (WBC) response

Platelet (thrombocyte) count

The client is receiving unfractionated heparin by infusion. Of which finding should the nurse notify the provider? A. Partial thromboplastin time (PTT) 60 seconds B. Platelets 32,000 C. White blood cells (WBCs) 11,000 D. Hemoglobin 12.2 g/dL

Platelets 32,000

The nurse is reviewing the medical record of a client admitted with heart failure. Which of the laboratory results warrants a call to the physician for further instructions? A. Calcium 8.5 mEq/L B. Potassium 3.0 mEq/L C. Magnesium 2.1 mEq/L D. International normalized ratio (INR) of 1.0

Potassium 3.0 mEq/L

A nurse is reviewing serum electrolytes and blood chemistry for a newly admitted client. Which result causes the greatest concern? A. Glucose: 97 mg/dL B. Magnesium: 2.1 mEq/L C. Potassium: 5.9 mEq/L D. Sodium: 143 mEq/L

Potassium: 5.9 mEq/L

What effects your stroke volume? (3)

Preload Afterload contractility

While monitoring a patient who has fluid overload, the nurse would be most concerned about which assessment finding? A. Pitting edema in the feet B. Neck vein distention C. Bounding pulse D. Presence of crackles in the lungs

Presence of crackles in the lungs

Which action does the RN delegate to unlicensed assistive personnel (UAP)? A. Drawing a partial thromboplastin time (PTT) from a saline lock on a client with a pulmonary embolism B. Performing a capillary fragility test to check vascular hemostatic function on a client with liver failure C. Referring for counseling a client with a daily alcohol consumption of 12 beers a day D. Reporting any bleeding noted when catheter care is given to a client with a history of hemophilia

Reporting any bleeding noted when catheter care is given to a client with a history of hemophilia

The nurse is caring for a client with peripheral arterial occlusive disease (PAD). For which symptoms should the nurse assess? A. Reproducible leg pain with exercise B. Unilateral swelling of affected leg C. Decreased pain when legs are elevated D. Pulse oximetry reading of 90%

Reproducible leg pain with exercise

A client is admitted with hypokalemia and skeletal muscle weakness. Which assessment does the nurse perform first? A. Blood pressure B. Pulse C. Respirations D. Temperature

Respirations

________ status should be assessed first in any patient who might have hypokalemia.

Respiratory Assess respiratory status at least every 2 hours because respiratory insufficiency is a major cause of death.

Which acid-base disturbance does the nurse anticipate the client with morbid obesity may develop? A. Metabolic acidosis B. Metabolic alkalosis C. Respiratory acidosis D. Respiratory alkalosis

Respiratory acidosis

An RN assesses multiple clients who are receiving transfusions of blood components. Which assessment indicates the need for the RN's immediate action? A. A partial thromboplastin time (PTT) that is 1.2 times normal in a client who received a transfusion of fresh frozen plasma (FFP). B. Respiratory rate of 36 on a client receiving red blood cells (RBCs). C. Sleepiness in a client who received diphenhydramine (Benadryl) as a premedication. D. Temperature of 99.1° F (37.3° C) for a client with a platelet transfusion

Respiratory rate of 36 on a client receiving red blood cells (RBCs).

A 77-year-old woman is brought to the emergency department by her family after she has had diarrhea for 3 days. The family tells the nurse that she has not been eating or drinking well, but that she has been taking her diuretics for congestive heart failure (CHF). Her laboratory results include a potassium level of 7.0 mEq/L. What is the primary goal of drug therapy for this client? A. Decreasing cardiac contractility and slowing the heart rate B. Elevating serum potassium levels to a safe range C. Maintaining proper diuresis and urine output D. Restoring fluid balance by controlling the causes of dehydration

Restoring fluid balance by controlling the causes of dehydration

A nurse is to administer packed red blood cells to a client. How does the nurse ensure proper client identification? A. Asks the client's name B. Checks the client's armband C. Reviews all information with another registered nurse D. Verifies the client's room number

Reviews all information with another registered nurse

What are the modifiable risk factors for coronary artery disease?

Risk factors that can be changed (smoking, obesity)

What are the nonmodifiable risk factors for CAD?

Risk factors that cannot be changed (age, gender, heredity, race)

Which client problem does the nurse set as the priority for the client experiencing chemotherapy-induced peripheral neuropathy? A. Potential for lack of understanding related to side effects of chemotherapy B. Risk for Injury related to sensory and motor deficits C. Potential for ineffective coping strategies related to loss of motor control D. Altered sexual function related to erectile dysfunction

Risk for Injury related to sensory and motor deficits

What is the pathway of conduction through the heart chambers?

SA node Atrial contractions AV node Bundle of HIS Bundle branches Purkinje fibers

CAUTION: A FLUID/MED with PH LESS THAN 5 PLUS MORE THAN 500 mOsm/L

SHOULD NOT BE ADMINISTERED BY VEIN

The nurse is caring for a client with hemodynamic monitoring. Right atrial pressure is 2 mm Hg. The nurse anticipates which request by the physician ? A. Saline infusion B. Morphine sulfate C. No treatment, continue monitoring D. Intravenous furosemide

Saline infusion

Which veins are used in coronary bipass surgery?

Saphenous and interal mammory

Which statement is true about the special needs of older adults receiving IV therapy? A. Placement of the catheter on the back of the client's dominant hand is preferred. B. Skin integrity can be compromised easily by the application of tape or dressings. C. To avoid rolling the veins, a greater angle of 25 degrees between the skin and the catheter will improve success with venipuncture. D. When the catheter is inserted into the forearm, excess hair should be shaved before insertion.

Skin integrity can be compromised easily by the application of tape or dressings.

A client who is suffering dyspnea on exertion and congestive heart failure will likely report which symptom during the health history? A. Fatigue B. Swelling of one leg C. Awakening at night to urinate D. Slow heart rate E. Brown discoloration to lower extremities

Slow heart rate

A 70-year-old with severe dehydration is ordered an infusion of an isotonic solution at 250 mL/hr through a midline IV catheter. After 2 hours, the nurse notes that the client has crackles throughout all lung fields. Which action does the nurse take first? A. Assesses the midline IV insertion site B. Has the client cough and deep breathe C. Notifies the health care provider about the crackles D. Slows the rate of the IV infusion

Slows the rate of the IV infusion

A nurse is teaching a group of teens about cancer risks. What does the nurse stress as the most important environmental risk for developing leukemia? A. Direct contact with others with leukemia B. Family history C. Living near high-voltage power lines D. Smoking cigarettes

Smoking cigarettes

There are 2 types of angina pectoris. What are they and what is the difference

Stable- goes away after exertion Unstable- unpredictable, nitro doesn't work, doesn't go away, and comes and goes without cause

WHat are the 2 major valve problems of the heart are what?

Stennosis Regurgitation

If you have fiberous plaques inside you arteries, what can happen to the vessels?

Stiffen, stenosis, can cause blood clots as well

When flushing a client's central line with normal saline, a nurse feels resistance. Which action does the nurse take first? A. Decreases the pressure being used to flush the line B. Obtains a 10-mL syringe and reattempts flushing the line C. Stops flushing and tries to aspirate blood from the line D. Uses "push-pull" pressure applied to the syringe while flushing the line

Stops flushing and tries to aspirate blood from the line

A client who is receiving a blood transfusion suddenly exclaims to the nurse, "I don't feel right!" What does the nurse do next? A. Calls the Rapid Response Team. B. Obtains vital signs and continues to monitor. C. Slows the infusion rate of the transfusion. D. Stops the transfusion.

Stops the transfusion.

The registered nurse would correct the nursing student when caring for a client with neutropenia secondary to chemotherapy in which circumstance? A. Student scrubs the hub of IV tubing before administering an antibiotic. B. Nurse overhears the student explaining to the client the importance of handwashing. C. Student teaches the client that symptoms of neutropenia include fatigue and weakness. D. The nurse observes the student providing oral hygiene and perineal care.

Student teaches the client that symptoms of neutropenia include fatigue and weakness.

The client who has undergone breast surgery is struggling with issues concerning her sexuality. What is the best way for the nurse to address the client's concerns? A. Allow the client to bring up the topic first. B. Remind the client to avoid sexual intercourse for 2 months after the surgery. C. Suggest that the client wear a bra during intercourse. D. Teach the client that birth control is a priority.

Suggest that the client wear a bra during intercourse.

Which of these clients who have just arrived in the emergency department should the nurse classify as emergent and needing immediate medical evaluation? A. The 60-year-old with venous insufficiency who has new-onset right calf pain and tenderness B. The 64-year-old with chronic venous ulcers who has a temperature of 100.1° F (37.8° C) C. The 69-year-old with a 40-pack-year cigarette history who is reporting foot numbness D. The 70-year-old with a history of diabetes who has "tearing" back pain and is diaphoretic

The 70-year-old with a history of diabetes who has "tearing" back pain and is diaphoretic

All of these client assignments have been made by the charge nurse. Which assignment would you question? A. The RN with 3 years of experience caring for a client with a pulmonary embolism (PE) who is receiving heparin therapy B. The LPN/LVN with 5 years of experience caring for a client with leg ulcers who is awaiting nursing home placement C. The RN with 8 years of experience caring for a client with peripheral arterial disease (PAD) and a total cholesterol of 390 D. The LPN/LVN with 20 years of experience caring for a client with a headache whose blood pressure is 210/150

The LPN/LVN with 20 years of experience caring for a client with a headache whose blood pressure is 210/150

Which of the following is essential to report to the provider when caring for a client with Raynaud's phenomenon? A. Nifedipine (Procardia) administration caused the blood pressure to change from 134/76 to 110/68. B. The client's extremity became white, then red temporarily. C. The affected extremity becomes purple and cold. D. The client states that the digits are painful when they are white.

The affected extremity becomes purple and cold.

Radiation dose

The amount of radiation absorbed by the tissue. The dose is always less than the exposure. Dose affected by intensity of exposure, duration of exposure, and closeness of the radiation source to the cells.

Exposure

The amount of radiation delivered to a tissue.

Four control systems play a major role in maintaining blood pressure:

The arterial baroreceptor system Regulation of body fluid volume The renin-angiotensin/aldosterone system Vascular autoregulation.

Fibrin molecules assemble together to form structures important in what?

The blood clotting process

All of this information is obtained by the nurse who is admitting a client for a coronary arteriogram. Which information is most important to report to the physician before the procedure begins? A. The client has had intermittent substernal chest pain for 6 months. B. The client develops wheezes and dyspnea after eating crab or lobster. C. The client reports that a previous arteriogram was negative for coronary artery disease. D. The client has peripheral vascular disease, and the dorsalis pedis pulses are difficult to palpate.

The client develops wheezes and dyspnea after eating crab or lobster.

The nurse is assigned to the following group of clients. Which of these clients should be assessed first? A. The client who had percutaneous transluminal angioplasty of the right femoral artery 30 minutes ago B. The client admitted with hypertensive crisis who has a nitroprusside (Nipride) drip and blood pressure (BP) of 149/80 C. The client with peripheral vascular disease who has a left leg ulcer draining purulent yellow fluid D. The client who had a right femoral-popliteal bypass 3 days ago and has ongoing edema of the foot

The client who had percutaneous transluminal angioplasty of the right femoral artery 30 minutes ago

What is cardioversion?

The delivery of a syncronized shotck to terminate atrial or ventricular tachyarrhythmias.

What is pulse deficit?

The difference between the apical pulse and the radial pulse

The nurse encourages the client to increase his fluid intake after a cardiac catheterization for which reason? A. NPO status will cause the client to be thirsty. B. The dye causes an osmotic diuresis. C. The dye contains a heavy sodium load. D. The pedal pulses will be more easily palpable.

The dye causes an osmotic diuresis.

Why would you tell someone that was overweight and had an MI to condition?

The goal would be to lose weight, strengthen the heart, reduce blood pressure, help with diabetes, increase the blood flow to the heart, lower cholesterol levels, decreses arterial stiffening

Why is iron an essential part of hemoglobin?

The heme part of each hemoglobin molecules need a molecule of iron to be able to transport up to four molecules of oxygen

The third line of defense against wide changes in body fluid pH.

The kidneys

Red Blood Cells (Erythrocytes)

The largest proportion of blood cells. Mature RBCs have no nucleus and have a biconcave disc shape

RADIATION THERAPY

The purpose of radiation therapy for cancer is to destroy cancer cells with minimal exposure of the normal cells to the damaging actions of radiation. The effects of radiation are seen only in the tissues in the path of the radiation beam, thus this type of therapy is a local treatment.

The second line of defense against pH changes.

The respiratory system

acute arterial occlusion

The sudden blockage of an artery, typically in the lower extremity, in the patient with chronic peripheral arterial disease.

Bone Marrow

The tissue responsible for blood formation

A nurse assessing a client's peripheral IV site obtains and documents information about it. Which assessment data indicate the need for immediate nursing intervention? A. Client states, "It really hurt when the nurse put the IV in." B. The vein feels hard and cordlike above the insertion site. C. Transparent dressing was changed 5 days ago. D. Tubing for the IV was last changed 72 hours ago.

The vein feels hard and cordlike above the insertion site.

Which assessment finding for a client with left ventricular failure who is receiving pulmonary artery pressure (PAP) monitoring indicates a need for rapid action by the nurse? A. The waveform shows that the catheter is continuously in the wedge position. B. Pulmonary artery wedge pressure (PAWP) is 12 to 14 mm Hg. C. The sterile dressing at the catheter insertion site has a bloody area that measures 1 cm. D. The chest x-ray shows the tip of the catheter in the pulmonary artery.

The waveform shows that the catheter is continuously in the wedge position.

A client is scheduled for a bone marrow aspiration. What does the client's nurse do before taking the client to the treatment room for the biopsy? A. Cleans the biopsy site with an antiseptic or povidone-iodine (Betadine) B. Holds the client's hand and asks about concerns C. Reviews the client's platelet (thrombocyte) count D. Verifies that the client has given informed consent

Verifies that the client has given informed consent

How "MUST" HYPERTONIC INFUSATE be infused?

Via PICC OR CENTRAL LINE

A nurse is transfusing a unit of whole blood to a client when the health care provider requests the following: "Furosemide (Lasix) 20 mg IV push." What does the nurse do? A. Adds Lasix to the normal saline that is infusing with the blood B. Administers Lasix to the client intramuscularly (IM) C. Piggy-backs Lasix into the infusing blood D. Waits until the transfusion has been completed to administer Lasix.

Waits until the transfusion has been completed to administer Lasix.

Which teaching should the nurse include for a client with peripheral arterial disease (PAD)? A. Elevate your legs above heart level to prevent swelling. B. Inspect your legs daily for brownish discoloration around the ankle. C. Walk to the point of leg pain, then rest, resuming when pain stops. D. Apply a heating pad to the legs if they feel cold.

Walk to the point of leg pain, then rest, resuming when pain stops.

Eosinophil

Weak phagocytic action Releases vasocative amines during allergic reactions

When caring for a client with cachexia, the nurse expects to note which symptom? A. Weight loss B. Anemia C. Bleeding tendencies D. Motor deficits

Weight loss

A nurse is caring for a client with neutropenia. Which clinical manifestation indicates that an infection is present or should be ruled out? A. Coughing and deep breathing B. Evidence of pus C. Fever of 102o F or higher D. Wheezes or crackles

Wheezes or crackles

Atherosclerosis,

a type of arteriosclerosis, involves the formation of plaque within the arterial wall and is the leading risk factor for cardiovascular disease.

The nurse is educating a group of women about the differences in symptoms of MI in men versus those in women. Which information should be included? A. Men do not tend to report chest pain. B. Men are more likely than women to die after MI. C. Men more than women tend to deny the importance of symptoms. D. Women may experience extreme fatigue and dizziness as sole symptoms.

Women may experience extreme fatigue and dizziness as sole symptoms.

Renal compensation results when

a healthy kidney works to correct for changes in blood pH that occur when the respiratory system is either overwhelmed or is not healthy.

Aldosterone

a hormone secreted by the adrenal cortex whenever sodium levels in the extracellular fluid (ECF) are decreased. It prevents both water and sodium loss. When it is secreted, it acts on the kidney nephrons, triggering them to reabsorb sodium and water from the urine back into the blood. This action increases blood osmolarity and blood volume. It prevents excessive kidney excretion of sodium. It also helps prevent blood potassium levels from becoming too high.

Body fluid pH

a measure of the body fluid's free hydrogen ion level

Therefore a change in the amount of plasma phosphorus results in an equal and opposite change in the amount of plasma ______ (and vice versa).

calcium

The ideal radiation dose is one that can

can kill the cancer cells with an acceptable level of damage to normal tissues.

Patients with severe hypermagnesemia are in grave danger of ________

cardiac arrest.

The IV route is used because MgSO4 ___________.

causes pain and tissue damage when injected IM.

normal chromosomes Euploidy

cells have 23 chromosomes which is correct for human beings.

environmental carcinogens are

chemical, physical, or viral agents that cause cancer.

Acid-base balance is regulated by ?

chemical, respiratory, and kidney actions

Statins reduce

cholesterol synthesis in the liver and increase clearance of LDL-C from the blood.

malignant- abnormal chromosomes- aneuploidy

chromosomes are lost, gained, or broken; cells can have more or less than 23 chromosomes; may have broken and rearranged chromosomes.

cancer classification

classified by the type of tissue from which they arise, or biologic behavior, anatomic site, and degree of differentiation.

grading

classifies cellular aspects of the cancer; needed because some cancer cells are more malignant than others. varying aggressiveness and sensitivity to treatment.

staging

classifies clinical aspects of the cancer; determines the exact location of the cancer and its degree of metastasis at diagnosis. stage influences therapy,

Sepsis or septicemia

condition in which organisms enter the bloodstream (bloodstream infection [BSI]) and can result in septic shock, a life-threatening condition. Patients with cancer are at risk for infection and sepsis because their white blood cell counts are often low and immune function is impaired.

progression

continued change of a cancer, making it more malignant over time.

Tumor Lysis Syndrome

large numbers of tumor cells are destroyed rapidly. Their intracellular contents, including potassium and purines (DNA components), are released into the bloodstream faster than the body can eliminate them Unlike other oncologic emergencies, TLS is a positive sign that cancer treatment is effective. Severe or untreated TLS can cause tissue damage and death. Serum potassium levels can increase to the point of hyperkalemia, causing cardiac dysfunction

EXTRAVASATION:

leakage of fluid into tissue surrounding a vein

Gynecomastia

literally means "female breasts" and is usually a benign condition of breast enlargement in men. However, it can be a result of a primary cancer such as lung or testicular cancer.

Common sites of metastasis for Colorectal Cancer

liver, lymph, adjacent structures

malignant- anaplasia

lose the specific appearance of their parent cells. becomes smaller and rounded. many types of cancer cells look alike and this make it harder to diagnose type of cancer.

Respiratory compensation occurs through the

lungs, usually to correct for acid-base imbalances from metabolic problems.

Interventions for respiratory acidosis aim to

maintain a patent airway and enhance gas exchange

Polycythemia vera is a cancer with three major hallmarks:

massive production of RBCs, excessive leukocyte production, and excessive production of platelets.

Brachytherapy

means "short" or "close" therapy, with the radiation source in direct, continuous contact with the tumor tissues for a specific period of time.

Acids are normally formed in the body as a result of

metabolism and incomplete breakdown of glucose and fats.

Liquids with a pH of 7.0 are ?

neutral—they have a free hydrogen ion level in which the number and strength of acids and bases are equal.


Conjuntos de estudio relacionados

Pharm Ad Chronic Disease, Pain, and Disability

View Set

CHEMISTRY - UNIT 2: STARTING THE INVESTIGATION: HOW TO IDENTIFY ELEMENTS, COMPOUNDS, AND MIXTURES USING CHEMICAL AND PHYSICAL PROPERTIES TO IDENTIFY SUBSTANCES (11th grade)

View Set

Chapter 2: Build-in Data Types for Programming? Part 2

View Set

Epi Final Cowan 2020 Quiz Questions

View Set